Этого треда уже нет.
Это копия, сохраненная 24 ноября 2018 года.

Скачать тред: только с превью, с превью и прикрепленными файлами.
Второй вариант может долго скачиваться. Файлы будут только в живых или недавно утонувших тредах. Подробнее

Если вам полезен архив М.Двача, пожертвуйте на оплату сервера.
Тред тупых вопросов №91 TSIOLKOVSKY EDITION 404988 В конец треда | Веб
Тред вопросов о жизни, Вселенной и всем таком.

Спрашиваем то, за что в других местах выдают путёвку в биореактор. Здесь анонимные ученые мирового уровня критически рассмотрят любые гениальные идеи и нарисованные в Paint схемы.

Предыдущий тут: >>401314 (OP)
https://2ch.hk/spc/res/401314.html (М)

Q: Можно быстрее?
A: Можно упасть в пузырь альбукерке, наса уже почти надула его.

Q: Я начитался охуительных историй от СМИ, че делать, нам жопа?
A: Да, тебе жопа, можешь сгонять в зогач, порашу или ньюсач или куда оттуда пошлют. К историям от СМИ относись с опаской.

Q: Что будет с человеком в вакууме без скафандра / если он упадет на черную дыру / попробует ступить на поверхность газового гиганта/солнца?
A: Он умрёт.

Q: Почему бы не привязать ракету к воздушному шару или стартовать с горы?
A: Космос - это не как высоко, а как быстро, большая часть энергии ракеты уходит на разгон вбок.
Подробнее тут https://what-if.xkcd.com/58/ (английский) https://chtoes.li/orbital-speed/ (перевод)
2 405048
>>04988 (OP)
Мне с 3-ьей крипотно чет, че делать? Открываю нулевую, а оно на меня таращится, пздц
3 405066
>>05048
Это всего лишь слияние двух небольших шариков в пару километров диаметром,
В нем нету ничего злого, но оно
пугает. Но после просмотра вы
почувствуете радость, и в
дальнейшем всё в вашей жизни
будет хорошо!

Ваш друг.
4 405092
Могут ли быть современные представления о чд, нейтронных звёздах, квазарах и подобном ошибочными?
5 405096
>>05092
Могут.
6 405102
>>05066
Анон говорит что на превью похоже на два глаза. От самой гифки только эпилептики просрутся.
image.png121 Кб, 464x494
7 405103
>>05102
Ну глаза и глаза, что ж такого-то.
8 405113
>>05096
Ничего себе, честный ответ на пейсаче. Не ожидал такого. Жму руку.
9 405115
>>05113
Вся квантовая механика была ошибкой. Декаденты 20х годов 20 века не могли придумать чего-то стоящего.
badumtss.png413 Кб, 783x776
10 405117
>>05115
Маня, плес.
11 405119
>>05117
По существу есть возразить?
12 405122
Что происходит со второй ступенью ракеты после отделения полезной нагрузки? Например у Falcon 9
13 405123
>>05122
Обычно остаётся болтаться в том же месте, фалкон по возможности направляет ступень назад на землю или куда нибудь подальше.
14 405129
>>05123

>фалкон по возможности направляет


Направляет или нет? Этих вторых ступеней должно там болтаться уже тысячи. Они ж сами не упадут
15 405130
>>05129
Направляет если остаётся топливо после выведения.
А в целом да, там прилично ступеней болтается.
Но некоторые сами падают.
16 405138
>>05122
полазь любопытства ради
http://stuffin.space/
17 405141
почему на мкс нет НИГЕРОВ?
18 405143
>>05141

>почему на мкс нет НИГЕРОВ?


В википедии есть 400 ссылок на американских астронавтов и где-то 20 астронавтов-негров.
То есть американец на МКС может быть ниггером с шансом около 5% и сейчас не свезло.
19 405145
>>05143

>сейчас


Думаю наса поэксперементировала и успокоилась. Одно дело нигеров насильно пихать в фильмы на роль бога/тора/иисуса/гарри поттера и другое пускать гамадрила на станцию стоимостью сотню миллионов тысяч баксов
20 405146
>>05145
В рамках стыковок шаттлов они точно бывали на МКС и даже выполняли там какие-то работы, но были ли они частью длительных экипажей, которые на МКС оставались - это мне каопать уже влом.
21 405154
Против углеродного шовинизма есть серьезные аргументы?
22 405169
>>05154
Спроси у шовинистов в /ро, хотя этот >>05145
может ответить
23 405170
>>05119
Как можно возразить по существу на беспруфный вскукарек, а, зелень?

Единственное содержательное утверждение у тебя:

>Вся квантовая механика была ошибкой


очевидно ошибочно само по себе. На минуточку, предсказания той же квантовой электродинамики подтверждаются в экспериментах с огромной точностью:
https://en.wikipedia.org/wiki/Precision_tests_of_QED
1.PNG3 Кб, 216x105
24 405305
>>05138
Спасибо за залипалово
мимоанон

Заодно хочу поинтересоваться, что это за шрапнель такая?
http://stuffin.space/?search=2017-042
http://stuffin.space/?search=2018-004
http://stuffin.space/?search=2018-015

Гугл нашел информацию про первый пуск, что-то про >70 спутников за раз и всё такое, но читать стены английского текста лень. Может, кто объяснит вкратце суть, а то это больше похоже на разбрасывание мусора, чем на научные исследования.

И последнее. Что такое TBA (пикрил)?
Тут https://acronyms.thefreedictionary.com/TBA ничего подходящего не нашел.
25 405314
>>05170

>подтверждаются в экспериментах


жидовские выдумки ненужны
(Автор этого поста был предупрежден.)
26 405382
Спейсач, реквестирую годные книги по небесной механике
(KSP не предлагать)
27 405387
>>05382
Левантовского уже прочитал?
28 405394
>>05305
Я привык, что TBA=to be announced.
Возможно, создатели надеются уточнить тип объекта и со временем обновить эту информацию.
29 405400
>>05387
А с чего лучше у него начать? Пытался одну прочитать, но показалась слишком сложной
30 405404
>>05400
Элементарное изложение сложно?
Ну тогда может реально лучше кербач?
31 405407
>>05404
эх, ладно, буду пытаться осилять
32 405411
>>05305
2017-042

>14 июля 2017 года в 09:36 мск с площадки 31 космодрома БАЙКОНУР запланирован пуск ракеты-носителя «Союз-2.1а» с космической головной частью в составе разгонного блока «Фрегат», космического аппарата (КА) «Канопус-В-ИК» и 72-х малых КА попутной нагрузки. Разгонный блок «Фрегат» обеспечит выведение КА «Канопус-В-ИК» и попутных малых спутников на три различные орбиты с последующим сведением с орбиты разгонного блока. Миссия такой высокой сложности будет реализована впервые в истории запуска малых космических аппаратов.



Скандальный рекордный пуск же, там кучу кубсатов положили, до сих пор терки идут, кто виноват.
https://www.roscosmos.ru/23686/

2018-004

>India's Polar Satellite Launch Vehicle, in its forty second flight (PSLV-C40), successfully launched the 710 kg Cartosat-2 Series Satellite for earth observation and 30 co-passenger satellites together weighing about 613 kg at lift-off.


https://www.isro.gov.in/launcher/pslv-c40-cartosat-2-series-satellite-mission

>2018-015


>China launched a Long March 2D rocket from the Jiuquan Satellite Launch Center on Friday, carrying a group of seven satellites from China, Italy, Argentina and Denmark including an innovative experiment to study the ionospheric precursors of Earthquakes to evaluate whether forecasting of strong Earthquakes will be possible from an orbital vantage point.


http://spaceflight101.com/long-march-2d-launches-zhangheng-1/

Юзай разные тематические ресурсы, там по этим индексам NSSDC ID https://ru.wikipedia.org/wiki/NSSDC_ID можно найти любую информацию.
33 405413
>>05394
>>05411

> Скандальный рекордный пуск же


Не следил за космосом в то время, да и сейчас не особо, прошло мимо ¯\_(ツ)_/¯

Спасибо!
загруженное.jpg5 Кб, 250x201
35 405416
>>05414
Ебануться, чесслово.
14093018956190s.jpg8 Кб, 166x200
36 405469
>>05314

>Автор этого поста был предупрежден

А почему, собственно, не батут? 37 405473
Вот Елон Муск сажает свои ракеты на землю. Но для этого используются тяжелые опоры, которые, вместе с топливом для посадки и рулями, снижают дельту ракеты больше чем в два раза. Так почему не избавиться от тяжелых опор и не сажать ракету на батут? Почему никто не догадался? Куда подать заявку на патент?
38 405476
>>05473

>снижают дельту ракеты больше чем в два раза


Откуда инфа?

>Так почему не избавиться от тяжелых опор и не сажать ракету на батут? Почему никто не догадался? Куда подать заявку на патент?


Потому, что не в два раза.
Ты еще предложи струнами ловить по бокам чтоб не накренялась.
39 405483
>>05476
Сейчас не найду статью на гиктаймсе, но вроде как примерно в два раза, если сажать на платформу в океане, если сажать на землю, то еще больше. Это косвенно подтверждается тем, что ракета по массе почти как "Протон", а грузы таскает уровня "Союза", что-то тяжелее уже только в одноразовом варианте.
40 405485
>>05483
Цифр у меня нет, так что спорить не возьмусь, но батут это реально омск.
41 405494
>>05473
Из чего ты предлагаешь сделать огнеупорный, выдерживающий падающие с неба ~30т, диаметром более трёх метров батут?

Ладно, сделал. Как будешь стабилизировать упавшую на него ракету? Земля/баржа жёсткая, в отличие от батута.
42 405496
>>05494
Так тормозить двигателями как при обычной посадке, просто оттормозившись падать на батут. И никаких опор не нужно.
43 405499
>>05496
На вопросы-то ты не ответил.

Вот упала ракета на батут, прыг-прыг и она уже летит с него в землю, дальше пикрил. Как фиксить будем?
44 405501
>>05496
>>05499
Ещё одно покормлю.
Предлагаешь избавиться от опор, а чем их заменить? Поверхность натянутого батута не сильно мягче земли будет, так что сила столкновения примерно равна. Садиться на сопла двигателей а-ля семейка Керманов - так себе затея, если хочешь потом использовать их ещё раз.

Полагаю, Муск и Ко. всё же не случайно сделали

> тяжёлые опоры


, а не маленькие палочки и батут.
45 405504
>>05501
Отправить Стивена ловить ракеты, очевидно же. Батут делаем в виде воронки.
Cnu2Cf-USW8.jpg354 Кб, 2405x1497
46 405513
Доставьте фото жилого отсека Союзов первых серий (не М и не Т).

Хочу посмотреть, правда ли там была деревянная столешница.
20-1.jpg117 Кб, 1200x748
48 405527
>>05520

Не она, это уже Союз-Т или позднее.
sage 49 405533
>>05527
Фото из музея National Space Centre
https://spacecentre.co.uk/blog-post/soyuz-spacecraft/
50 405539
>>05520
>>05533
Там же люк наверху, АЛЛО. Это Союз-Т.

В Великобритании выставлена восстановленная копия беспилотного Союз 7К-ОК с банкой вместо Иглы. Это фото не оттуда.
51 405541
>>05539
Вернее с Иглой, но в другой конфигурации
53 405543
>>05542

круто, и правда похоже! Не массив, наверное, и с рейлингами, но всё же - столешница!

Спасибо.
54 405544
Там кстати немец какой-то книгу написал про Балашову, уже вышла. Такой-то прикладной промдизайн на острие ретрофутуризма!
262db30f35.png42 Кб, 458x145
55 405552
Известно, что у самых больших звёзд почти весь объём состоит из очень разрежённого вещества, почти вакуума. Но в то же время оно ионизировано и непрозрачно. Но на каких расстояниях эта непрозрачность проявляется? Насколько ерохин и сычёв должны отлететь друг от друга, чтобы не увидеть?
56 405585
Когда фоточки с EHT подвезут?
57 405606
Могут ли на внеземных объектах быть какие-то недоступные для землян ресурсы? Я краем уха что-то слышал про самородки золота на астероидах и богатые руды других металлов, насколько это правда? Может ли астероид быть с настолько высоким содержанием какого-нибудь осмия, что его теоретически было бы интересно распотрошить и отправить на Землю? Или другое вещество, которое на Земле в естественном виде не существует (или почти не существует), но представляет интерес для промышленности? Возможны условия для формирования таких астероидов?
58 405609
>>05606
Фактически, всё золото, кобальт, железо, марганец, молибден, никель, осмий, палладий, платина, рений, родий и рутений, которые сейчас добываются из верхних слоёв Земли, являются остатками астероидов, упавших на Землю во время ранней метеоритной бомбардировки, когда после остывания коры на планету обрушилось огромное количество астероидного материала. Из-за большой массы более 4 млрд лет назад на Земле начала происходить дифференциация недр, в результате чего большинство тяжёлых элементов под действием гравитации опустилось к ядру планеты, поэтому кора оказалась обеднённой тяжёлыми элементами. А на большинстве астероидов из-за незначительной массы никогда не происходила дифференциация недр и все химические элементы распределены в них более равномерно.
59 405611
>>05609

>все химические элементы распределены в них более равномерно.


Самородки-то будут или это всего лишь значит, что надо будет отсеять из руды не 99.9999% шлама, а целых 99.99%?
Снимок.JPG97 Кб, 752x415
60 405612
>>05611

>будут

1528280404041-1949586420.jpg624 Кб, 723x1458
61 405614
>>05611
Вот эта хуйня весит 2,7*1019 кг, из них 90% железо и никель.
62 405618
>>05609
Спасибо
63 405632
Течёт ли кровь у самок из щелей в невесомости?
image.png157 Кб, 640x480
64 405639
>>05609
Это не совсем так. Несмотря на тяжесть материала некоторые элементы (литофильные) остались в коре. А сидерофильные (железо, кобальт, никель и платиновые с золотом) потонули в ядро, их и стоит искать в астероидах пока не откроем рилейтед.
65 405642
>>05632
Естественно.
66 405656
Правда что космонавты суеверные и у них есть дни(числа) в которые запуски никогда не намечаются?

Насколько я понял сейчас ракеты в космос не запускает только ленивый. Собственно вопрос зачем? и благодаря чему такая доступность технологий?

Как они не поврезались ещё в космосе?
67 405663
>>05656
Насчёт дней хуй знает, но традиций дохуя.
Запускают мало кто на самом деле.
Не сталкиваются потому что там огромные расстояния.
68 405666
>>05656

>космонавты суеверные


как и многие другие люди, имеющие профессии связанные с повышенными рисками

> у них есть дни(числа) в которые запуски никогда не намечаются


Я читал, что такое выжигали каленным железом еще при СССР. Суеверия - суевериями, но на работу это не должно сказываться. Числа 13 никто ну или делают вид не боится.

>сейчас ракеты в космос не запускает только ленивый


На данный момент всего полдесятка стран и столько же частников, остальные делают небольшие перделки-свистелки с пуском раз в несколько лет

>благодаря чему такая доступность технологий?


Накопленный опыт дидами, интернет, открытость технологий, развитая система привлечения инвестиций, поддержка государств. Мб контроль еще меньше чем 20-30 лет назад, палестинцы, арабы и корейцы уже давно имеют что хотели.

>Как они не поврезались ещё в космосе?


Космос, даже ближний, это много-много в степени много кубических километров ничего. Принимается ряд мер по упорядочению объектов в космосе. Например, на высоты орбиты МКС запрещено что-либо запускать чтобы исключить даже невероятно маленькую возможность столкновения. На некоторых высоких орбитах, в которых сконцентрированы спутники, международное сообщество заставляет в конце срока работы уводить аппараты на т.н. мусорные орбиты. Ну и "мусорить" поменьше. Запускать по 80 студенческих спутников там нельзя. Что летает низко, довольно быстро падает на Землю и сгорает. Все это дело контролируется с Земли, создается 3д-модель всех объектов, просчитываются безопасные траектории. Перед запуском на орбиту необходимо получить разрешение страны-хозяйки, а та в свою очередь отвечает перед ООН.
69 405668
>>05642
И как они ее собирают? Там же нет гравитации, кровь вниз не потечёт.
70 405670
>>05668
Сельдь, плес
71 405672
>>05668
Во-первых, циклы в невесомости слегка меняются (за пруфами не полезу, может меня поправят).
Во-вторых, прокладки и тампоны по-прежнему работают.
В-третьих для истечения жидкостей из тела не нужна гравитация, они текут большей частью из-за перистальтики.
В-четвертых, вот видео как справляются на орбите с истекающими жидкостями
https://www.youtube.com/watch?v=P36xhtpw0Lg
72 405680
С МКС улетают на своем союзе или предыдущем?
73 405684
>>05680
На предыдущем. Неча Союзу торчать на орбите слишком долго дабы не пошло что не так.
74 405688
>>04988 (OP)
Почему при образовании звезд, из газопылевого облака протопланетный диск превращается именно диск?
Почему на земле раньше было сильно теплее, хотя солнце было слабее?
75 405695
>>05680
На своем, конечно, там же кресла даже индивидуально подогнаны. Полгода союз висит пристыкованный и готовый к отлету, чтобы в случае чего можно было быстро по местам рассесться и покинуть МКС.
76 405697
>>05684

>>05695

Спасибо!
77 405698
>>05697
Нахуй ты спасибо говоришь, не ты спрашивал.
78 405699
>>05698
Мне приятно.
79 405710
>>05695
Вообще-то они скручивают ложементы со своего Союза и перетаскивают в предыдущий.
80 405711
>>05710

>Вообще-то они скручивают ложементы со своего Союза и перетаскивают в предыдущий.


хуяси
81 405717
>>05710
Еще колеса скручивают у бесхозных припаркованных союзов и ставят их на кирпичи.
82 405726
>>04988 (OP)
Скажи мне, зачем нужна МКС? Что там такого можно исследовать? Зачем держать там по полгода людей?
И почему нет чёрных на МКС?
83 405727
>>05726
Платина.
Тебе - незачем. Для фундаментальной науки и исследований тема, ибо где ты еще найдешь микрограви тацию?
84 405729
А правда что космонавты постоянно саботируют систему переработки мочи чтобы не пить мочу и не позориться?
85 405732
>>05727
Есть падающий ИЛ-76 например.
Тогда по-другому спрошу. Нахуя она такая огромная?
Ну сделали ли бы две банки: одна спать и срать, другая работать. Для пары человек достаточно. Какого хуя они такую мразь там налепили?
86 405733
>>05729
Она для технических нужд.
87 405734
>>05729
Нет. Рекуперация воды из мочи идет на бытовые нужды. Пьют свежую с планеты.

>>05732

>Есть падающий ИЛ-76 например.


Может ли он падать трое суток? То-то и оно.

>Нахуя она такая огромная?


Так вышло.

>Для пары человек достаточно. Какого хуя они такую мразь там налепили?


Чтобы работало не пару человек а полдюжины
88 405736
>>05734

> Чтобы работало не пару человек а полдюжины


Зачем?
89 405740
>>05736
Чтоб работать, вестимо
90 405749
>>05736
Станции на пару человек это пройденный этап, ниче интересного там нет.
91 405751
>>05749
Скучновато, картишки не разложить, нардишки не погамать. А в компании всяко веселее.
92 405754
>>05751
Да, на скайлэбе даже как то раз космонавты взбунтовались.
fukken lold8.jpg83 Кб, 682x600
93 405755
>>05754
Проиграл.
95 405757
>>05756
Я помню этот случай, просто звучало удачно так, что я и проиграл.
96 405761
>>05656

>есть дни(числа) в которые запуски никогда не намечаются?


24 октября потому что 24 октября 1960 (говорят, что не летают, я не проверял).
97 405762
>>05761
Хз насчет пилотируемых, их не так много было, чтобы все дни гарантировано заполнить, а так летали:

два в 62, 63, 64, 67, два в 69, 74, 78, 80, 85, 89, 96, 2012

И к Марсу АМС пускали, и Восходы с Зенитами, и военные аппараты, и баллистические ракеты.
Стикер512x512
98 405765
>>05656
Я наверняка уверен, что запуски НИКОГДА не намечаются на прошлое время.
99 405845
Космонавт Людмила - правда?
100 406002
Какой максимальный импульс на высококипящих топливох без скатывания в совсем ужасные вещи типа боранов?
101 406008
А где срут в банках без дополнительных отсеков типа союза и лунного Аполлона?
В белом футуристчином драконе срать прямо на людях? Там хоть сортир будет или вручную говно из жопы доставать придется?
102 406009
>>06008
Там место специальное за шторкой.
104 406034
>>06008

>банках без дополнительных отсеков типа союза


У союза есть отсек, в БО толкан размещен.

> лунного Аполлона


В пакетик.

>В белом футуристчином драконе срать прямо на людях? Там хоть сортир будет


Вроде во всех новых капсулах гальюны предусмотрены.
105 406053
Не могу понять, почему найдя самую дальнюю галактику на расстоянии 13 млрд лет, учёные решили, что это и есть возраст Вселенной?
106 406055
>>06053
Потому, что она самая молодая?
107 406062
>>06053
Это не дальняя. Наблюдаемая вселенная 42 млрд вроде, но это из-за расширения вселенной.
108 406064
>>06062
Но постой! Возраст же 13 млрд лет! Откуда 42?
109 406065
>>06064
Расширялась быстрее скорости света.
110 406071
В крошечных галактиках на тысячу звёзд есть дыра в центре?
111 406074
>>06053

>на расстоянии 13 млрд лет



Не на расстоянии 13 млрд световых лет, а свет от галактики летел к нам 13 миллиардов лет, это разные вещи. Пока фотоны летели, пространство успело нихуево так расшириться, и прямо сейчас расстояние до самых далеких галактик составляет более 30 миллиардов световых лет.

Ну и галактики не так важны, как реликтовый микроволновый фон, оставшийся с реально ранних времен (первый миллион лет после Большого взрыва), вот по нему точнее меряют.
112 406081
>>05845
Нет, хуйня.
113 406130
>>06065
Но быстрее света ничего нет.
114 406131
>>06130
А это иной принцип. Движутся не галактики, а расширяется пространство между ними. Естественный Альбукерке короче.
115 406137
Ебанный стыд...
Во-первых, Алькубьерре.
Во-вторых, не упасть, а создавать вокруг корабля изнутри (иначе кина не будет).
В-третьих, НАСА искривляет пространство на десятимиллионную часть, контролируя это сверхточными интерферометрами, до самого варп-привода здесь - как до Антарктиды раком.
116 406157
>>06131
Я так понимаю, пространство внутри галактик тоже расширяется, но их не распидарашивает, потому что гравитация подсуетилась?
>>06137
Розового покемона этому хлопцу
117 406158
>>06157
Вообще всё расширяется, вся вселенная. Ты тоже.
118 406160
>>06157
Хуй знает почему они не рвутся, темная материя/энергия или прочая мутная хуйня.
119 406164
>>06158

>Ты тоже.


Так почему я из тредов вытекаю...
120 406166
>>06160
А можешь уточнить?
121 406196
>>06157

>не распидорашивает


Ну возьми гандон кингсайз длиной в один километр. Поставь на нём вдоль длинной оси две отметки на расстоянии одного нанометра одна от другой. Растяни гандон в два раза. Как думаешь, на сколько увеличится расстояние между сделанными тобой ранее отметками?
122 406198
>>06166
На данный момент никто не может.
123 406199
>>06196
В джва раза?
124 406212
>>06196
Хуевая аналогия, все не так.

Более приближенная к реальности — на гондон положили два грузика на небольшом расстоянии друг от друга, и сцепили их ниткой. Начали тянуть гондон, нитка растянулась на 0,00001%, сила упругости нитки стала больше «отрицательной силы» расширения пространства, и грузики перестали удаляться друг от друга.
Сколько бы мы гондон ни тянули, нитка не порвется и грузики не сдвинутся с места, потому что растягивающая сила зависит только от расстояния между грузиками, а оно не увеличивается.

Так же и во всей вселенной, расширение пространства не распидорашивает ни атомы, ни галактики, потому что связывающие их силы на порядки сильнее крошечного растяжения. А так как расширение пространства пропорционально только расстоянию и ничему больше, то эта растягивающая сила остается постоянной, и не распидарасит их и в будущем.

Но на действительно крупных масштабах уровня сверхскоплений галактик и выше, слабые гравитационные силы уже не могут преодолеть расширение пространства, и скопления галактик разлетаются в стороны друг от друга, причем скорость разлета, пропорциональная расстоянию, со временем все увеличивается.
125 406223
>>06212

>со временем все увеличивается


Вот тут спиздел, а так все верно
126 406238
Вот у нас есть 2 равноценных, 1 килограмовых куска быстро распадающегося топлива для ритега. Один кусок мы раскладываем по 100 грамм каждый, в 10 контейнеров. А второй кусок запихиваем в контейнер целиком.
Спустя н времени мы собираем 1 ритег вновь и сравниваем радиоактивность этих кусков.
Будет ли разница, в какую пользу и насколько сильно ?
127 406243
>>06238
Смотря какое топливо, для стронция не будет вообще никакой разницы, а для плутония
скорее всего совсем чуточку более радиоактивны будут 100-граммовые куски, потому что осколки деления в килограммовом куске могли инициировать больше дальнейших реакций деления и чуть быстрее уработать оставшийся плутоний. Но разница там будет совсем маленькая.
128 406245
>>06212
Я прост для колхозанов попытался в объяснение.
Ведь что там сраные пара светолет протяжённости галактик, если эта протяжённость изменяется в масштабах, минимум, тридцати миллиардов световых лет...
То есть, наверное, на Планковскую длину за лолиард лет. Математик-кун, скальуцлируй, плез.
1526575711725.jpg35 Кб, 221x246
129 406246
Почему бы не преобразовывать солнечную энергию в...
Ведь тогда можно было бы...
В общем, космический аппарат в космосе с практически неисчерпаемым источником энергии.
130 406247
>>06246
Солнечные панели изобрел?
131 406248
>>06247
Ну я имел ввиду что солнечные понели не очень сами по себе и не получили широкое распространение.
132 406250
>>06248
Потому что для ловли энергии нужны огромные площади, потому что они ненадёжны, потому что чем дальше от солнца - тем хуже светит.
133 406252
>>06250
Можно лампочками освещать или кострами.
134 406253
>>06250
А почему нет температурных энерговыделителей? Чтобы кинул в сахару его и добывал фри энергию?
135 406254
>>06248
Ну а вообще они на каждой хуйне стоят, но много от них не получишь.
Хотя даун на них до цереры долетел.
136 406255
>>06253
Есть топливные элементы.
Но они кончаются, а солнце нет.
mentalnoeporno51.gif3 Мб, 520x520
137 406264
У меня вопрос. Берём некую солнечную систему, в которой нет карланов и гигантов, но есть много землеподобных планет. Все планеты расположили свои орбиты так близко друг к другу, что у них у всех постоянно происходит орбитальная рокировка подобно взаимодействию Януса и Эпиметея у Сатурна. Только эти два меняются только друг с другом, а в данной системе ВСЕ землеподобные каменные планеты меняют орбиты от самой близкой к звезде да самой дальней.
Мне очень интересно, как может сложиться цикличность погоды, климата и даже какой может быть жизнь на одной или ан всех этих планетах.
138 406265
>>06253
Низя. Либо аккумулируй тепло, римляне с зеркальным щитом, баня, пар, турбина, вот это всё, а это очень дорого, сложно и кпд херовый, либо всякие термопары, а это тоже дорого и сложно, и кпд ниже чем у фотоэлементов.
139 406266
>>06265
Неужели нет никакого хайфхака на халявную энергию, ведь столько вокруг всего...
Я слыша лчто локхид мартин что-то там пилить революционное.
140 406270
>>06266
Чем тебе солнечные панели не нравятся? Покрыть Сахару, вырастить боевых гмо-верблюдов для охраны периметра от мародеров, хватит на весь черный континент.

Ну хочешь, можно выращивать микрофлору, которая будет жрать #something и давать спирт на выходе, собственно в этом направлении яйцеголовые и продвигаются.
141 406271
>>06270

>Ну хочешь, можно выращивать микрофлору, которая будет жрать #something и давать спирт на выходе, собственно в этом направлении яйцеголовые и продвигаются


Интересно, первый раз слышу. А на спирте что делать?
V-2kvn.jpg53 Кб, 566x690
142 406273
>>06271
Ракеты, спейсач же.
143 406337
>>06253
Коассически энергия черпается из разницы потенциалов. Вот ты нагрел уде всё - и чо блчть? Где ПРОЦЕСС, при течении которого можно спиздить, взять откат на разнице?
145 406362
>>06359

>совокупность явлений, в которых разница температур создаёт электрический потенциал


>разница температур



Начала термодинамики у тебя там не открыли еще, что ли? Невозможно создать полезную энергию просто из тепла, нужен еще и холодильник.
146 406365
>>06362
М - Магнетизм
147 406387
>>06365
Ты - хлебушек.
>>06337
>>06362
148 406392
>>06212
Только ты забыл, что с увеличением расстояния сила упругости нитки увеличивается, а гравитационное притяжение ослабевает. Так что, если скорость расширения увеличивается, то рано или поздно распидарасит и голактеке и плонеты и даже атомы и даже аллаха. ПОС она такая
149 406393
>>06264
Ты же понимаешь, что разница между орбитами

>Януса и Эпиметея


разница с пару сотен метров?
150 406395
Солнечный зонд Паркера подлетит к Солнцу на 6 млн километров от него. От Земли до Солнца 150 млн. Значит ли это, что он будет получать в (150/6)^2 больше излучения, чем на нашей орбите? В 625 раз? Сколько там ватт на кв.метр спутники получают на орбите Земли?
151 406399
>>06393

>Ты же понимаешь, что разница между орбитами


>>Януса и Эпиметея


>разница с пару сотен метров?


Серьёзно что ли?
152 406402
>>06399

>Большая полуось151 460 км


>Большая полуось151 410 км


>Серьёзно что ли?


Нет, конечно
153 406403
>>06399
50 км между орбитами, но по факту они не сближаются менее чем на 10000 км.
154 406405
Как быстро и без смс вычислить угловой размер наблюдаемого объекта, если известно расстояние до него и размер?
155 406413
>>06405
Арктангенс (размер/расстояние), но для малых углов можно просто размер/расстояние взять, разницы почти нет. Не забудь только из радиан в секунды дуги перевести, если надо.
156 406416
>>06413
А если сильной точности и не требуется, то можно сразу в угловые секунды через формулу размер/расстояние*200000.
157 406427
>>06416
А хорда, опирающаяся на угол меньший в 2 раза, тоже получается примерно меньше в 2 раза, правильно же? Опять же для малых углов, если длина хорды с википедии = 2 x R x sin(a).

Т.е. угловой размер (для малых углов) прямо пропорционален размеру объекта?

Если угловой размер Луны 32 минуты, то угловой размер булыгана с радиусом Луны/10 на таком же расстоянии будет в 10 раз меньше, около 3 минут?
158 406433
>>06402
>>06403

> 50 км между орбитами, но по факту они не сближаются менее чем на 10000 км.


При том, что они малыши ~150 и ~100 км. А если землеподобные планеты? Думаю, так гравитационного влияния планет будет достаточно, чтобы при настижении более низокоорбитальной планетой более высокоорбитальной, наблюдаемый эффект произойдёт и с двумя планетами, заставляя их меняться местами. И, честно говоря, до сих пор не вижу обоснованных причин, почему не может быть системы, в которой планеты, настигая друг друга, меняются орбитами. Более того, чтобы смена орбита у планеты в такой системе проходила только с одной и той же планетой, а не ходила бы ближе-дальше.
159 406436
>>06245

>пара светолет протяжённости галактик


...
160 406453
>>04988 (OP)
Какой климат лучше: с низкими температурами или с высокими?

Вот у нас на Земле в умеренном климатическом поясе температура изменяется от ~-30 зимой до ~+30 летом при условии что комфортные температуры для человеков составляют от ~+10 до ~+30 не смотря на это с помощью технологических достижений мы более менее выдерживаем зимний период, а что если бы было наоборот? То есть зимой температура максимум падает до ~+10 а летом возрастает до ~+70 было бы человечеству легче переживать горячий вариант или наоборот сложнее? Наблюдали бы мы ситуацию вроде: Зимой всё цветёт, а летом всё вянет от жары и т.п.*
161 406454
>>06453
Вопрос личных предпочтений
162 406457
>>06453
Такс... ну тут у нас получилось бы следующее:
полярные территории стали бы умеренно холодными, их можно было бы заселять, на экваторе в этом случае была бы совсем йоба печка, скорее всего из за этого вменяемо даже не получилось бы плавать между двумя умеренными полюсами, так что путешествовали бы самолётами. На экваториальной суше тоже ловить не чего разве что солнечных батарей наставить, но вопрос кто в таких условиях их ставить будет.. При таких температурах океаны бы испарялись намного активнее, облачный покров на планете был бы в несколько раз плотнее, дожди были бы гораздо чаще и на большей территории, в целом атмосфера бы даже стала немного плотнее и насыщена влагой - это бы несколько сгладило климат между холодными частями и жаркими, но в целом было бы весьма душновато.. Для комфортной жизни пришлось бы поближе к полюсам засесть, в более жарких частях планеты в летнее время пришлось бы обмазываться радиаторами, охладителями и прочим в целом более сложным технологическим оборудованием нежели обогреватели. Основная жизнь кипела бы в ночное и утреннее\вечернее время, днём бы все старались свалить от дневного света куда подальше исключением разве что была бы облачная погода или если на территорию загонит холодные воздушные массы с полюсов. Касательно воздушных масс.. В настолько нагретой и чуть более плотной атмосфере йоба циклоны с антициклонами возникали бы на порядок чаще, всякие катаклизмы в лице ураганных ветров, смерчей и т.п. тоже возникали бы чаще и на большей территории, в общем было бы НЕПРИЯТНО . Как бы приличная часть людей совсем на полюса не перебрались, предпочитая контрить низкие температуры обогревателями зато при этом более спокойную часть атмосферы. Растительность была бы в большей степени адаптированная к высоким температурам, но в периоды лета вполне могла бы сбрасывать листву, разбрасывать семена до более холодных времён и т.п.

Кстати тут надо ещё смотря чем вообще вызваны высокие температуры, если в несколько раз более плотной чем земная атмосферой, то умеренные температуры могли бы установиться в целом повсюду с минимальными сезонными и суточными колебаниями, если речь о более горячей звезде, то тут может быть НЕПРИЯТНО больше не от температуры а от возросшей доли ультрафиолетового излучения в спектре звезды + более горячие звёзды заразы такие живут меньше. Если же речь о том, чтобы быть по ближе к звезде то тут аккуратно надо, а то и тидально локнуть может, ну или как минимум сильно замедлить вращение планеты вызвав более острые суточные колебания. Если же нагрев вызван воздействием приливных сил от газового гиганта\парной планеты\спутника на умеренно низкой орбите, то тут вообще могут появиться регулярные йоба планетотрясения что испортят жизнь по больше чем радиация и температура, в случае газового гиганта же можно и радиации от него дофига нахватать. Ну примерно так.
162 406457
>>06453
Такс... ну тут у нас получилось бы следующее:
полярные территории стали бы умеренно холодными, их можно было бы заселять, на экваторе в этом случае была бы совсем йоба печка, скорее всего из за этого вменяемо даже не получилось бы плавать между двумя умеренными полюсами, так что путешествовали бы самолётами. На экваториальной суше тоже ловить не чего разве что солнечных батарей наставить, но вопрос кто в таких условиях их ставить будет.. При таких температурах океаны бы испарялись намного активнее, облачный покров на планете был бы в несколько раз плотнее, дожди были бы гораздо чаще и на большей территории, в целом атмосфера бы даже стала немного плотнее и насыщена влагой - это бы несколько сгладило климат между холодными частями и жаркими, но в целом было бы весьма душновато.. Для комфортной жизни пришлось бы поближе к полюсам засесть, в более жарких частях планеты в летнее время пришлось бы обмазываться радиаторами, охладителями и прочим в целом более сложным технологическим оборудованием нежели обогреватели. Основная жизнь кипела бы в ночное и утреннее\вечернее время, днём бы все старались свалить от дневного света куда подальше исключением разве что была бы облачная погода или если на территорию загонит холодные воздушные массы с полюсов. Касательно воздушных масс.. В настолько нагретой и чуть более плотной атмосфере йоба циклоны с антициклонами возникали бы на порядок чаще, всякие катаклизмы в лице ураганных ветров, смерчей и т.п. тоже возникали бы чаще и на большей территории, в общем было бы НЕПРИЯТНО . Как бы приличная часть людей совсем на полюса не перебрались, предпочитая контрить низкие температуры обогревателями зато при этом более спокойную часть атмосферы. Растительность была бы в большей степени адаптированная к высоким температурам, но в периоды лета вполне могла бы сбрасывать листву, разбрасывать семена до более холодных времён и т.п.

Кстати тут надо ещё смотря чем вообще вызваны высокие температуры, если в несколько раз более плотной чем земная атмосферой, то умеренные температуры могли бы установиться в целом повсюду с минимальными сезонными и суточными колебаниями, если речь о более горячей звезде, то тут может быть НЕПРИЯТНО больше не от температуры а от возросшей доли ультрафиолетового излучения в спектре звезды + более горячие звёзды заразы такие живут меньше. Если же речь о том, чтобы быть по ближе к звезде то тут аккуратно надо, а то и тидально локнуть может, ну или как минимум сильно замедлить вращение планеты вызвав более острые суточные колебания. Если же нагрев вызван воздействием приливных сил от газового гиганта\парной планеты\спутника на умеренно низкой орбите, то тут вообще могут появиться регулярные йоба планетотрясения что испортят жизнь по больше чем радиация и температура, в случае газового гиганта же можно и радиации от него дофига нахватать. Ну примерно так.
163 406463
164 406464
Во-первых, Солнце - это планета! Во-вторых, откройте энциклопедию по физике, найдите "Солнце". Обратите внимание на ускорение свободного падения на поверхности Солнца =257 м/с^2 То есть Солнце, согласно научным данным - чёрная дыра. Тогда как происходит выброс плазмы при таком ускорении? На каком расстоянии от поверхности Солнца надо разместить теннисный мяч, чтобы Солнце по теории Эйнштейна исчезло? Температура на Солнце - это фазовое состояние атомной структуры. Есть фотографии НЛО (или материального тела), который спокойно летает вблизи поверхности Солнца.
165 406466
>>06464
Ну пиздец это не то что не космоопера и даже не космофентези это уже воистину мир бреда и абсурда, 10 из 10 \b\++...
166 406518
Господа, у меня параноя или последние скажем так 10 лет в атмосферу планеты слишком часто начали падать астероиды?
Возможно это связано с распространением камер, просто стали чаще на записи попадать регулярные для планеты события
167 406524
>>06518

>у меня параноя


this. Астероиды уже 65 миллионов лет не падали
168 406526
>>06524
Абсолютно точный и абсолютно бесполезный ответ.
Все время забываю что надо искать ответы самому а не спрашивать анонимных уёбков в сети.
Спасибо что напомнили, добра вам )
169 406527
>>06526
Иди с миром и не трать наше время больше. И таблетки прими.
170 406535
>>06524
Уебок, съеби с этой доски
171 406537
>>06524
Але, маня, астероиды падают каждый год, или у тебя есть какое-то свое собственное определение?
172 406544
>>06524
>>06526
>>06535
>>06537
Девчата, не ссорьтесь, только тред засрали почём зря. Астероид это шелупонь от 1 метра и до здоровенной хуйни под 1000 километров в диаметре.
173 406545
Хочу зайти в ракетное моделирование с перспективой поднять пепелац до 100км+ Тех опыта за плечами нет, в каком порядке курить литературу?

Есть место под полигон, без людей, там даже и ПВО то нет, даже никто и не узнает.
174 406547
>>06545
Начни с административного кодекса (если ты из этой страны). Пиротехнические составы в военном деле, пороходелие, теория летательного аппарата. Кури форумы карамельщиков - полезно. Чуть пониже есть /spc prgrm тред, там немного литературы вбрасывали
175 406548
>>06547

А если у меня есть знакомые мужики которые могут подогнать цистерну жидкого кислорода?
176 406551
>>06548
Жидкостный двигатель очень сложно сделать в гараже, только петарды. А им кислород не очень нужен.
177 406552
>>06548
Ты всё равно не сможешь его хранить. И учти, что ЖРД на порядок(ки) сложнее твердотопливного. (Хотя в вышеозначенном треде говорили, что АК не ограничивает запуски ЖРД)
178 406554
>>06551
Ну тащемта можно запилить гибридный двигатель а-ля Рутан, но это будет всё равно сложнее, чем чисто твердотопливный
179 406555
>>06551

>Жидкостный двигатель очень сложно сделать в гараже



Че там сложного если не мудрить с нагнетателем и нагнетать батареечкой как одни умные люди? 150 атмосфер и не надо, 50 нагнать - уже неплохо а болванку камеры и сопла васьки-токарщики сделают.
180 406556
>>06555

>нагнетать батареечкой


Щито?
181 406557
>>06556

Ракета электрон.
182 406560
>>06556
Для подачи в камеру сгорания топлива и окислителя используется два электронасосных агрегата (ЭНА) мощностью 37 кВт[2][4][5]. Поскольку каждый компонент подаётся собственным насосом, расход и соотношение компонентов регулируются изменением расхода насосов, а не с помощью дросселя и регулятора как в традиционном ЖРД с использованием турбонасосного агрегата[1]. Такая схема управления двигателем впервые использована на действующей ракете-носителе[6].

Каждый ЭНА приводится в действие бесщёточным электродвигателем переменного тока, питающимся от установленных на ступени литий-полимерных аккумуляторных батарей через инвертор. Максимальная частота вращения центробежного насоса составляет 40 000 об/мин, что позволяет изменять давление в топливной магистрали от 0,2 до 20 МПа (от 2 до 200 кгс/см²
183 406561
>>06557
Мог бы и сам рассказать. Вообще, аккумуляторы для обслуживания нужд ЖРД - зашквар. Может, топливные элементы вышли бы компактнее, но всё равно многократно тяжелее турбонасоса
184 406564
>>06561

>Вообще, аккумуляторы для обслуживания нужд ЖРД - зашквар.



https://www.youtube.com/watch?v=eg5234BOED8

Летает вроде.
185 406565
>>06564
С лёгким ТНА летало бы быстрее, выше, сильнее. Хотя если делать дёшево и сердито - сойдёт. Интересно, где они взяли дмгатели на 40000 оборотов
videoplayback.webm20,8 Мб, webm,
640x360, 4:58
187 406600
>>06555
Да вообще без проблем, хули там собирать
188 406627
>>06600

попилы-откаты
189 406629
>>06561
Тем не менее рокетлеб пока единственные, кто получил у керосинового двигателя УИ на уровне моря за 300 не закрытым циклом
190 406630
>>06600
Да, меня это очень забавляет. Казалось бы, первые российские частники, пилящие ракеты. А у них же на сайте в новостях

> Несмотря на финансовые и административные сложности мы уверенно продвигаемся вперед к светлому космическому будущему для всего человечества!

191 406631
Да, кстати, насчёт ЖРД. Я тут посмотрел, у азота температура кипения -195°C, у ацетилена -83°C. Получается, что с помощью жидкого азота вполне можно получить жидкий ацетилен. Какие тут подводные?
192 406632
>>06631
Ты хоть представляешь себе, какие меры приходится принимать для того, чтобы даже газообразный ацетилен не бомбанул? Безо всяких причин, просто потому ч тоему так хочется. А уж о том, чтобы сжимать его и речи не идёт
193 406633
>>06632
Зачем сжимать? 25 атмосфер в стандартном сварочном баллоне должно хватить более чем. Если газообразный ацетилен пропускать через тонкую трубку, охлаждаемую ниже температуры кипения, то это должно сработать. Утечки это другой вопрос
194 406634
>>06633
Я накачу за тебя, когда ты взлетишь на воздух
195 406635
>>06634
Пусть это теоретически будет в атмосфере почти без кислорода, азотной допустим. Сам принцип то работать будет?
196 406638
Можно ли приспособить верхнюю ступень протона под ещё более опасное топливо?
197 406646
>>06638
новая топливная смесь = новый двигатель = новая ступень
198 406649
>>06646
Не всегда же.
199 406653
>>06630
А что ты хочешь, если можно набутылиться за сахар?
200 406654
>>06633
Нюфаня, ацетилен в сварочном баллоне находится не в сжатом, а в растворённом виде. Попробуй как-нибудь приподнять "пустой" ацетиленовый баллон, лолкек.
201 406658
>>06654
В сжатом, но с абсорбером.
202 406660
>>06561
Но ведь аккумуляторы это разновидность топливных элементов.
sage 203 406664
>>06629
Мечтай.
>>06649
А как иначе-то.
204 406665
>>06664
https://ru.wikipedia.org/wiki/Резерфорд_(ракетный_двигатель)?wprov=sfla1

> Удельный импульс Вакуум: 333 c


> Ур. моря: 303 c

205 406666
>>06664
Ну навскидку могу привести в пример двигатель агены.
sage 206 406668
>>06665

>ru.wikipedia.org


мда
>>06666
Поменяли смесь азотной кислоты и тетраоксида азота на смесь азотной кислоты и тетраоксида азота, ну.
207 406672
>>06666
Титан еще. Перевели с керосинки на НДМГ.
208 406687
Какой предел размеров и мощности для петард?
209 406692
>>06687
Самую большую вроде Aerojet делали, диаметром 260 дюймов (6,6 метров), и она даже кое-как работала на наземных тестах, но до полетов не дошло. Может, и больше можно, хуй знает.
210 406706
>>06660
Не совсем. Скорее сходство имеется с неперезаряжаемыми гальваническими элементами, но ТЭ имеют внешний источник реактивов а не ограничены запиханным в них при производстве количеством. Да и реактивы там совсем другие
211 406713
>>06254
Джуно, тащемто, тоже солнечные панели имеет, а он вокруг Юпитера летает, лел.
212 406726
>>06713
Хуевое сравнение, даун летал на ионниках с запиткой от панелей, а у джуно гептилка.
213 406760
Чем закончилась история Звезды Табби?
достроили они там свою теплицу?
214 406772
>>06760
Естественное происхождение наиболее вероятно.
215 406790
Насколько сложно организовать курилку на МКС?
216 406800
>>06790
Да в принципе несложно по сравнению c тем, что там уже есть. Но зачем?
357439677.jpg19 Кб, 268x265
217 406803
>>06800
Чтобы курить.
218 406804
А нельзя в прогрессе курить, например?
219 406813
>>06804
Он же махонький, вангую что там от собственного дыма через минуту прихуеешь.
220 406818
>>06813
>>06804
>>06790
В форточку кури
221 406822
>>06818
Артемьев жалуется, что у него от частой разгерметизации уши болят.
222 406825
>>06822
Ну так пускай ватку проспиртованную засунет.
223 406826
>>06825
Я когда ему твой совет передал, он эту ватку мне пообещал засунуть. Куда не сказал, но я в принципе догадываюсь.
224 406828
>>06822
В американский модуль иди
225 406831
>>06790

Ну дым понятно. Поставить вытяжку и пропускать через неё. А вот что с пеплом делать это сложнее вопрос.
226 406833
>>06831
Ну, если на МКС срут в пылесос, то и пепел в пылесос стряхивать решение очевидное.
227 406839
>>06726
Об ионниках вообще разговора не было.
228 406877
>>06772
Давай подробности
229 406885
>>06658
Ладно.

>...Допускать в ацетиленовых баллонах давление, значительно превышающее 25 кг/см2, нельзя по условиям безопасности. Вследствие этого пористую массу пропитывают ацетоном, который существенно повышает газовбираемость, так как является хорошим растворителем для ацетилена.

230 406889
В чем разница между гидростатическим равновесием и просто круглостью?
231 406890
>>06889

Если планета из однородной жидкости вращается, то она уже не будет шариком.
232 406918
Какие были планы на Энергию-Буран, Энергию-2 и Вулкан?
233 406920
>>06918
Размещение оружия в космосе.
234 406934
>>06918
В сущности-то не было.
235 406937
Если титану сделать содержание кислорода как на земле, там всё ебанет?
236 406959
>>06918
Смотрите, пиндосы челнок делают! Мы тоже хотим!
237 407038
>>06937
Нет, кислород сконденсируется и не сможет вступить в реакции при такой температуре и давлении.
Вилка.jpg82 Кб, 1024x768
238 407066
Тут недавно проскакивала новость, что Оппортьюнити перевели в ждущий режим, чтобы не тратил энергию, из-за пылевой бури, которая засрала ему солнечные панели. А после бури как он их чистить-то будет?
239 407067
>>07066
Ну вообще буря закрыла все небо и там темнее чем ночью, засерание вторично. В прошлый раз пыль сама сдулась, например.
240 407069
Кстати почему ему не приделали дворники или вентилятор?
241 407072
>>07069
У него миссия блять на 90 дней была рассчитана, кто ж мог подумать что он там 14 лет будет бури превозмогать.
interstellar-case-1024x578.jpg63 Кб, 1024x578
242 407073
А какова хуя черные дыры вращаются?

На их горизонте время для внешнего наблюдателя уже бесконечно замедлено, почему тогда жидовские ученые говорят, что какая-то хуйня внутри вращается на бешеных оборотах?
243 407077
>>07073
Извинись за жидовских ученых сначала, а потом получишь ответ.
244 407079
>>07073

>внешнего наблюдателя


А причем тут внешний наблюдатель-то вообще? Дыра вращается вне зависимости от его наличия или присутствия. Или "если я не вижу луну, значит ее нет"?

Дыра вращается потому что момент импульса сохраняется. Если она, например, получилась путем коллапса массивной звезды, то для начала представь, что звезда вращалась. Не очень быстро. Потом после коллапса часть звезды стала ЧД. И эта ее часть продолжила вращаться. А чо б ей не продолжить.
только выиграли даже ускорилась.

Как фигуристка ускоряет вращение, когда руки к сиськам поджимает.
245 407083
>>07079
Тогда почему нам есть какое-то дело до этого? Мы же внешний по отношению к черной дыре наблюдатель, и никакой эргосферы быть не должно.
246 407085
>>07083
Ну черная дыра не вечный объект же, ну, они испаряются когда-то. Когда-то какие-то эффекты из эргосферы могут возыметь наблюдаемый эффект, при каких-то условиях.
247 407086
>>07083

>Мы же внешний по отношению к черной дыре наблюдатель, и никакой эргосферы быть не должно


Блеать, я не понимаю как в твоей логике связаны наличие или отсутствие внешнего наблюдателя и наличие или отсутствие эргосферы.

Кто тебе сказал, что наблюдатель на что-то влияет? Эргосфера у вращающейся ЧД есть не потому за ней кто-то наблюдает или не наблюдает.
248 407088
Если вращение дыры неважно для окружающего мира, от чего зависит направления вращения аккреционного диска?
249 407089
Какая скорость объектов на границе наблюдаемой вселенной относительно нас?
250 407090
>>07088
Зависит от природы аккреционного диска. Если это остатки говна, которое вращалось от звезды, которая когда-то стала черной дырой, то это вращение будет преобладать. Но черные дыры ведут активную сексуальную жизнь в звездных парах, так что там может появиться такой партнер, который по ретроградке будет двигаться к анусу черной дырочки и потом с него одежду также будут по ретроградке снимать...
sage 251 407091
>>07088

>вращение дыры неважно


Говну всегда что-то "неважно". g:"метрика Керра"
252 407092
У нас есть человек в скафандре с рюкзаком. В рюкзаке находится некий массивный объект, который человек запускает в строго одном направлении придавая себе ускорение. Запускает он его среднестатистическим броском рукой, с поправкой на то что он очень точен, и кидает предметы в строго одном направлении, со строго одинаковой силой.
Как лучше всего придавать себе импульс если кидать по 1 грамными шариками или сразу оттолкнутся от 1 тонной конструкции?
253 407093
>>07092
Ну, чем выше скорость рабочего тела, тем эффективнее движение.
255 407101
>>07093
Ну и где будет выше скорость? Если оттолкнутся от 1000 кг тела или от 1 кг тела тысячу раз?
256 407123
>>07098
Если кому интересно, он после испытаний с видеорилейтед до сих пор в этой шахте лежит.
257 407149
>>06877
Там карочи плонита, оче большая плонита, прямо писец короче, и вокруг этой планеты оче очебельные йоба кольца, прямо огромнейшие до невозможности и они короче и закрывают свет, а ещё там оче толстый и богатый на астероиды пояс астероидов это ещё дополнительно форсит мемес отсутствие фотонов.
258 407159
>>07101
1000 от 1 кг, ты чего? Очевидно, что бросить 1 кг шарик получится с большей скоростью, импульс - масса х скорость. Анон выше же написал.
259 407161
>>07123
По этим фото не скажешь, что 4 человека должны друг другу на плечи встать чтобы сравнять размеры.
изображение.png2,8 Мб, 1920x1080
260 407166
Ой хватит мне сказки за вечность заяснять, я думаю ты жестянка даже число Грэма лет не проживёшь.
261 407167
>>07166

>Скакал углеродоущерб неосиливающий сохранить сложную структуру даже две сотни лет.



Как тебе постоянные процессы отмирания тканей и попыток пожрать тебя изнутри "симбиотической" микрофлоры?
262 407169
>>07167
Ай, это же не тот тред, глубочайше извиняюсь за каку.
263 407174
>>07073
Потому что момент импулса.
264 407195
Как работает магнитное поле? Появляется разница в заряде между двумя объектами, отрицательно заряженные частицы начинают перескакивать от "минуса" к "плюсу", правильно же? Если поставить стенку - частицы не перескакивают, всё, пизда полю.
Свет, радиоволны - поток определенных частиц. Поставил стенку - ничего не проходит.
Радиоактивное излучение - поставил стенку, ничего не проходит.

Что является "переносчиком" гравитационного взаимодействия? Школьная физика и википедия говорит, что сила гравитации подчиняется закону обратных квадратов, что говорит о том, что переносчиками гравитации являются какие-то частицы. Откуда берется энергия и такое количество "переносчиков" на это постоянное и бесконечное взаимодействие между всеми объектами и даже элементарными частицами? Почему эти частицы не взаимодействуют друг с другом, как такое вообще может быть? Почему вдруг от гравитации нельзя спрятаться, почему это вдруг сила гравитации не уменьшается при воздействии через другие объекты, такие как миллион тонн породы Земли между бухим в салат аноном в пятницу и ядром планеты?

Не надо кидать сюда ссылки на научные работы в 300 томов, если анон не может объяснить простым и доступным читателю языком - значит он сам нихуя не знает. Может мы все живем в Матрице, а?
265 407196
>>07195
Ну это бля гравитационные волны короче
266 407197
>>07159
Ну и что будет иметь большую скорость?
Запуск тысячи килограмовых шаров или запуск одного 1 тонного шара?
267 407198
>>07197
Какую скорость ты можешь придать шару в 1 грамм и в 1 тонну?
268 407204
>>07092
Тут хак в том, что ты будешь 10^6 раз бросать шарики в 1 грамм с ускорением в 30 м/с^2 и той же силой, с которой всего 1 раз оттолкнешь от себя 1 тонный шар с 0.1 м/с^2.

Если, например, заменить твою руку на манупилятор, причем сила затраченная на бросок будет строго пропорциональна разряду батареи и стоит задача бросить все шарики, то тогда сила приложенная к шарику в 1 грамм будет в 10^6 раз слабее и разницы никакой не будет.
269 407206
А вот стандартная пара кислород + керосин.

насколько падает КПД если в камере сгорания не 150 атм а 50? или вообще фокуса не происходит? или нужно менять пропорцию?
270 407208
У какой ракеты самая хуевая аэродинамика?
Ricerocket6.jpg31 Кб, 450x321
271 407209
>>07208
У ракеты с квадратным обтекателем, я полагаю.
272 407213
>>05092

Они скорее всего и есть ошибочные, так же как представления средневековых врачей или алхимиков.
273 407224
>>05141

От перегрузок на старте у них губы лопаются. антиквартная шутка из 80-х
274 407230
>>07196
Свет - это тоже волна, но свет не умеет проходить метр бетона. Почему гравитация может?
275 407236
>>07230
Ну нейтрино через что угодно проходят, например.
276 407266
>>07195

>Что является "переносчиком" гравитационного взаимодействия?


Ты это на борде спрашиваешь? Лучшие умы человечества, бьются над разгадкой уже столетие, коллайдер построили и ещё толком не поняли как и почему. Если узнаешь, что является переносчиком, то нобелевка, да какая нахуй нобелевка, вообще всё что угодно тебе будет обеспечено.
277 407271
>>07236
>>07266
Жидовская наука какая-то.

А как нейтрино проходит? Как оно?он? не сталкивается с хуллиардом других частиц по пути? И почему тот же фотон не может так же?

Почему они не теряют скорость во время огибания препятствий?
278 407281
>>07271
У тебя есть шанс начинать узнавать это с сентября, слушая учителя, а не пинеая хуи на двачах
279 407286
Анэны, есть ли фотографии марсоходов сделанные другими марсоходами?
281 407288
>>07286
Нет, какой смысл сажать их в одинаковое место?
282 407289
>>07287
Судя по гуглу, это селфи какого-то марсохода после какого-то бурения, но не фото марсохода сделанное другим марсоходом.
Есть трустори фото?
283 407290
>>07288
Шоб сделать красивое фото, очевидно же
284 407292
>>07287
Я чет не пойму где палка за которую держится развёрнутая на марсоход камера? Или уже технологии антигравитации\тонкого контроля магнитных полей освоены?
285 407293
>>07067

>В прошлый раз пыль сама сдулась, например.


И после этого атеистобляди смеют утверждать что Бога нет, ясно.
286 407294
>>07293
Его и нет тут у нас ТВЁРДАЯ реальность, так что только загоны физики в данном случае в лице ветра.
287 407295
>>07294
Ты действительно такой тупой и не понимаешь, что ветер в разреженной атмосфере Марса не мог случайно появиться в определенный момент?
Мда, ну ты и скептик.
288 407296
Чому чёрные дыры не могут увидеть, если вокруг них должны быть ебические аккреционные диски?
latest[1]275 Кб, 483x620
289 407297
>>07295
Будь реальность не твёрдой мы бы уже псионическую теорию изучали и в Покров лезли. Но вместо этого только на ракеты водой брызгаем.
290 407298
>>07297
Зачем споришь с религиозным дураком?
291 407299
>>07298
Да так, "просветить хотел".
292 407300
>>07296

Ну наблюдений рентгеновского излучения оттуда достаточно много.
293 407301
>>07290
Только аполлоны вроде садились рядом с сурвейором и делали фоточки, но это потому что сурвейоры должны были искать место посадки.
>>07292
Фото из многих кусков потому что.
295 407304
>>07301

> Только аполлоны вроде садились рядом с сурвейором и делали фоточки, но это потому что сурвейоры должны были искать место посадки.


А фото нет в открытом доступе?
Гуглю на ангельском, ничего не могу найти.
296 407306
>>07304
Бля, я тупой.
Я думал речь идет о Марсе, но на самом деле о Луне >>07303
297 407312
>>07296
Аккреционные диски тебе ничего не должны
298 407315
Че там кстати с фоточками чёрной дыры в центре? Я уже начинаю подозревать что это мошенничество.
299 407316
Что будет если завести на марс растения и микробы, толерантных к тамошней среде?
300 407324
>>07316
На марсе будут растения и бактерии, толерантные к его среде.
39.jpg32 Кб, 620x370
301 407326
>>07324
Не может быть
302 407344
>>07281

> есть шанс


Поздно уже, нету. Последняя надежда на двачи
303 407382
>>07344
Есть четыре фундаментальных взаимодействия — сильное, слабое, электромагнитное и гравитационное, причем каждое последующее слабее предыдущего на несколько порядков. Сильное и слабое еще и относятся к короткодействующим, т.е. убывают с расстоянием сильно быстрее, чем по закону обратных квадратов.

Чем в меньшем количестве взаимодействий участвует частица, и чем более слабы эти взаимодействия, в которых она участвует, тем более ей похуй на материю и тем свободнее она проходит через что угодно.

Нейтрино, например, участвует только в слабом и гравитационном взаимодействии, да еще и имеет ничтожно малую массу, поэтому ему практически полностью поебать на любое вещество и оно легко пролезет без всякого мыла прямо через атомное ядро. Слабое взаимодействие, как и следует из названия, не шибко сильное, и действует на очень малых расстояниях, а гравитация начинает заметно влиять на траекторию нейтрино только начиная с поистине астрономических масс.

Гипотетический переносчик гравитации, если и существует, то скорее всего не участвует вообще ни в каком взаимодействии, помимо гравитационного, проходит насквозь вообще через все, и задетектить его вообще хуй знает как.
304 407395
>>07382
А какое отношение к этому имеют гравитационные волны и ЛИГО?
305 407396
>>07395
Никакого, это не частицы и не переносчики гравитации.
306 407397
Не знаю тупой вопрос или нет, в общем как попасть на работу в обсерваторию?
Где лучше учиться, на какой специальности. Можно ли попасть в обсерваторию без соответсвующего образования? Какие там нужны специалисты, непосредственно астрономы, электронщики, радиофизики и т. д. Если у меня уже есть высшее образование не по профилю (в данном случае инженер ПГС), можно ли в таком случае туда попасть? Сложно ли это сделать?
307 407400
>>07382
Спасибо
308 407434
>>07312
Они мне излучение должны.
309 407470
>>07434
Чё, и долговая расписка есть?
310 407475
>>07470
Ага, зверенная Максвеллом и Эйнштейном.
311 407489
>>07286
Если бы такое фото было, то это бы значило то, что ученые отправили два марсохода, стоимостью пару лярдов долларов, исследовать одно и то же место, что в принципе является глупостью.
312 407496
>>07382
Слабое и электромагнитное - одно и то же электрослабое взаимодействие, т е не четыре, а три
313 407497
>>07496
Так можно и до теории всего и единого взаимодействия допиздеться, обычно рассматривают все же четыре.
314 407498
>>07496
Олдфаг в треде, все в сингулярность!
315 407505
Помогите опознать блок маневровых двигателей плиз
https://twitter.com/runnymonkey/status/1008323221874233344
316 407506
>>07505
Больше всего похоже на двигатели Блока Е, а не Г, пикрилейтед.
http://www.yuzhnoye.com/company/history/block_e.html
317 407507
>>07505
На нём же ж написано - 11Д71 - блок ДУ СОЗ блока Д.
318 407512
>>07507
>>07506
Спасибо большое.
319 407513
Что произойдет с планетой, если ее масса (не объем) изменится хотя бы на 1%?
320 407514
>>07507
Там этому мужику уже кто-то в твитторе твой ответ передал если что.
321 407517
>>07513

Особо ничего. А как ты себе это представляешь?
322 407523
>>07517
Ну, сместится ли она с орбиты, изменится ли ее скорость вращения вокруг своей оси (или вокруг солнца)?
323 407526
>>07523

Это зависит от изменения импульса и момента импульса. Если импульс останется прежним, то станет чуть ближе летать к светилу и чуть медленнее вращаться.
324 407529
>>04988 (OP)
Есть ли в космосе ещё какие йобы, типа черной дыры? Чтоб необычно так было и захватывающе.
237190611f92a8233136570b9918d9b32e82cdd39800.jpg25 Кб, 320x240
325 407530
>>07526
Спасибо, анон
326 407532
>>07529
Кефир
327 407539
>>07529

Тёмная материя, тёмная энергия, квазары, скопления чёрных дыр, гамма-всплески, двойные звёзды. Войды.
328 407540
>>07539

>Войды


Можно подробнее.
329 407542
>>07540
Огромные пространства где ниче нет.
330 407543
>>07540
Очень дохуя пустоты. На пикиведии же:
https://ru.wikipedia.org/wiki/Войд
331 407546
Зачем у посадочных аппаратов с Венеры были тормозные парашюты?
332 407549
>>07546
Чтобы подольше пробыть в верхних слоях и насобирать инфы о том как там оно, насколько я помню.
igry-novosti-2016-04-30-035.jpg25 Кб, 644x362
333 407647
>>07540
Ставь купол на как можно большее количество вражин, прожимай мом и манту и еби в ротешник всех кого можешь
334 407684
а большой взрыв на самом деле не взрыв потому, что не детонировал?
335 407685
>>07684
Это был Большой Пук.
336 407712
>>07684
Не всегда взрыв сопровождается детонацией
15292551059320.jpg127 Кб, 658x960
337 407716
Кто сказал, что раньше Земля была меньше? За счёт чего же она тогда сейчас выпучилась?
Если так, то как же тогда гравитация работала?
338 407718
Каким образом Арес 5 должен был тащить 180 тонн?
СЛС по факту мощнее Ареса, более эффективные маршевые двигатели и ускорители, но в максимальной конфигурации обещают 130-150 тонн
339 407719
>>07718
RS-68 раза в полтора мощнее RS-25, на верхней ступени J-2 в десять раз мощнее одного RL-10.
340 407720
>>07719
Но у rs25 больше УИ.
А какой импульс у rl 10? Я не нашел
341 407722
>>07720
Компенсируется большей массой топлива. У RL-10 импульс 465, лучший среди химических, против 440 вроде у J-2X.
342 407727
>>07716
Кто тебе сказал что Земля была меньше?
343 407728
>>07727
Научпопные фильмы про Пангею
344 407730
>>07722

>лучший среди химических


Ты забыл про КВД-1
345 407731
>>07716
Да так же она работала, в пределах долей процентов, большее влияние Луна оказывала, так как ближе была.
346 407732
>>07730
Нет ты.
Импульс 462 против 465, про применение и говорить нечего.
347 407734
>>07728
Если ты про ловлю астероидов то там прирост небольшой.
Если про Тею то это совсем древние времена и тогда Пангеи не было.
DfwgDLDUYAAgB4U.jpg418 Кб, 1200x1197
348 407742
>>07734
Нет, там про то, что, мол, раньше, Луна была ближе, сутки длились меньше, месяцы были меньше, но их было больше, угол наклона Земли был нулевой и гравитация была слабее, поэтому, типа, динозавры и могли быть такими большими, не коллапсировать под собственным весом.
349 407744
>>07742
Во времена динозавров сутки и правда были на час-два короче, но система Земля-Луна мало чем отличалась от современной, и земная гравитация была ровно такая же.

Динозавры и сейчас могли бы быть такими большими, и ноги бы под ними не сломались, это все брехня. Перенесенные в сегодняшний день динозавры бы, конечно, сдохли, но вовсе не из-за выросшей силы тяжести.
15176285164500.png23 Кб, 1106x458
350 407754
Я ньюфаг, поясните за плюсы такой вот вращающейся станции.
351 407757
>>07754
Искусственная гравитация.
352 407759
Супчика единственные кто делает прожоги ступени. Вопрос почему и как скоро это станет повсеместным?
353 407760
>>07759
Фикс супчик на Спусикс
354 407761
>>07754

Компактная станция с искусственной гравитацией и небольшой скоростью вращения при этом.
355 407828
Вода уникальный элемент? Я слышал что она несжимаемая или что-то такое, поясните.
356 407829
>>07828
Плотность жидкого состояния выше чем твердого, это все.
357 407830
>>07829
Я не понял.
Да и разве не твердлое тело более плотное?
358 407839
359 407840
>>07830
У всех остальных - да, твердое плотнее жидкого. У воды нет. Но это не значит что ее нельзя сжать, в газовых гигантах есть разные формы льда с огромной температурой.
360 407841
>>07757
>>07761
Спасибо :3
361 407845
>>07840

>У всех остальных - да, твердое плотнее жидкого.



Кремний, галлий, германий, висмут, сурьма, церий и тысячи разных химических соединений с тобой не согласны, расширение при затвердевании довольно необычное, но нихуя не уникальное явление.
ч (6).jpg60 Кб, 500x500
362 407877
Меня озарило, и решил поискать гуглплее приложение, которое при наведении на реальное небо, все звезды подписывало. Я поставил то что сверху, но внутри оказалась только интерактивная карта, которая вроде как не работает в режиме виртуальной реальности. Мой реквест вообще осуществим?
363 407878
>>07877
Heavens above norm rabotaet
364 407879
>>07878
Там вроде только спутники.
365 407880
>>07877
Star chart
366 407881
>>07879
Планеты, если увеличить созвездия и звёзды.
367 407884
Как поведёт себя солнце, если всё его вещество перемешать до одинаковой плотности, температуры и состава?
368 407885

> Температура межгалактического газа составляет порядка десяти миллионов градусов.


это если туда положить предмет, он будет постепенно "разъедаться"? насколько быстро? или нас всех переживёт ?
369 407888
>>07884
И внезапно надобно уточнить до каких конкретно плотности, температуры и состава. Предположительно ты подразумеваешь параметры верхних слоев. Если до температуры в 6к К и плотности очень разреженного ионизированного газа, то обратный гравитационный коллапс очень большого газового шарика. А вот с составом уже сложнее.
370 407896
>>04988 (OP)
Меня мучал вопрос наша звездная система уже движется с какой то скоростью, но скрость света постоянна итд. Как мы можем прям уверенно говорить о том что вселенная расширяется? а не тут наблюдаем оптические иллюзии?
И в целом с какой скоростью движемся в млечном пути?
371 407898
>>07742
а Луна в конце концо улетит? или что
У меня есть только информация вроде, что если прокрутить путь вращение Луны то окажется что Луна никогда не была частью земли итп, но это знания полученные рандомно в школе.
Нигде немогу найти ответ на вопрос луна улетит с орбиты земли?
372 407901
>>07888
Нет, именно до средней. Например, адиабатически разделить солнце на кубометры и рандомно расположить их внутри солнечной сферы (на незанятых местах), повторить n раз. И охуевать с того, чё будет.
373 407903
Пацаны! У меня вопрос. Почему черные дыры охуительно сильно не светятся? Я думаю что это работает так: при подлете к черной дыре время замедляется, так? Чем ближе к горизонту событий, тем сильнее замедляется, верно?
В итоге из-за этого мы даже не можем увидеть как вещество звезды, которую разорвало притяжением ЧД упадет в черную дыру тк чем, ближе подлетает она к горизонту событий, тем сильнее время замедляется, тоесть там около горизонта должно висеть дохуища вещества, которое ОЧЕНЬ медленно падает в ЧД, так медленно, что мы даже не должны это видеть, верно? И получается, что там дохуя вещества скопилось еще очень-очень-очень старых звезд, верно? Почему тогда она не светится пиздец сильно?
Безымянный.png24 Кб, 1243x648
374 407904
И вот еще вопрос. На каком расстоянии от черной дыры начинает замедлятся время? Из-за того что время замедляется свет до нас летил как бы медленнее, верно? Сейчас мы все время смотрим в прошлое, так ведь? Говорят мол звезда от нас в 10-ти миллионах световых лет от нас, значит мы видим ее такой какой она была 10 миллионов световых лет назад. Настоящего не существует вообще, лол.
Так вот и теперь представим что там рядом со звездой черная дыра. Я не знаю на каком расстоянии ее поставить, что бы она воздействовать начала. Так вот! Будет ли как-то это влиять на то какой мы ее будем видеть? Или все так же 10 миллионов лет назад?
КосмонавтОлегАртемьевпоказалкаквыглядитспутникпролетающийми[...].mp41,4 Мб, mp4,
640x640, 0:24
375 407908
А можно ли спутничек летящий на МКС вручную оттолкнуть на другую траекторию? Выйти так в скафандре и ручками или ножками обнять, а потом выкинуть?
376 407909
>>07903

> время замедляется, так?


Это пиздёшь. Просто теоретический сладкий хлебушек в голове у математухов. Замедляется взаимодействие материи. Времени не существует как и расстояния, это всё абстракция, чтобы обезьянке было легче банан с пальмы доставать.
377 407910
>>07909
Ты опять выходишь на связь, ебанутый?
378 407911
>>07903
время замедляется же для вещества которое падает туда, а не для нас. но эффекты должны быть очень интересные при тщательном наблюдении.
379 407913
>>07903
Грубо говоря, вторая космическая у чд выше чем скорость света, то есть из нее ничего не может улететь. Чтобы мы могли видеть что-то, фотоны и прочие частицы должны улететь из дыры и долететь до наблюдателя, но им не хватает скорости.
Вещество там в основном от первоначальной звёзды и немного говна с округи.
>>07904
Радиус Шварцшильда. Для воздействия на звезду она должна быть очень близко, звёзда такого не выдержит и достаточно быстро "засосется" дырой.
Но если на безопасном расстоянии, возможно гравитационное линзирование если они встанут на линию, если такое найти можно пиздато разглядеть далёкие звезды.
380 407914
>>07908
с мкс ручками запускают
https://www.youtube.com/watch?v=lC0OOTg5XJ0
381 407915
>>07754
Намотает жи на камень. Или будут кружить с центром посередине. Я не ебу как там внатуре, но интуитивно так. Нужно видимо играться с массами и направлением тяги.
382 407916
>>07910

>Ты опять выходишь на связь


Да, а что? Поясни за время, что за сущность и как она отделима от планковской длинны и дискретно тактового пространства? Есть просто постоянное движение, а время это количество тактов смещения.
383 407917
>>07916
Сам придумал?
384 407919
>>07877
Star Walk 2.
Не уверен, что он actually фотографирует небо, а не выводит тебе полупрозрачную карту с привязкой к GPS и положению телефона, но вроде звёзды совпадали довольно неплохо, когда я по-пьяни с друзьями смотрел на небо в телескоп.
385 407923
>>07917
По аналогии с компьютером.
386 407929
>>07915
Чтобы намотало на камень, центр масс системы должен быть под поверхностью камня, а на картинке он снаружи.
VKnpQoxxfxw.jpg756 Кб, 1638x2048
387 407933
Анон, помоги с мыслёй. В общем, представил себе фантастику и думаю над ней.
Значит ты, анон, летил на космическом корабле к другим звёзда, въёбся в тебя камушек и твою бандуру распидорасило и тебя выбросило на стабильную более-менее круговую орбиту вокруг неизвестной планеты в неизвестной тебе системе. Но в Штабе эту систему наверняка знают. У тебя есть вот-вот подохнущий передатчик, который выдержит только одну передачу. Тебе нужно масимально чётко и точно описать характеристики планеты, звезды и других планет системы, смотря из окошка/иллюминатора/кабины. И тут я подумал "А это вообще возможно?"
По сути, ты не знаешь ни размера планеты, ни массы планеты, ни расстояния до звезды - ничего. Ты только знаешь, за сколько времени пролетаешь от точки А до точки А. Допустим, ты летишь строго по экватору планеты.. И ты можешь засечь только время между пролётом утреннего экватора в первый и во второй раз, и это ещё при условии, что планета не вращается, потому что тогда и и сам терминатор сдвигается.
В общем, анон, ты обречён или как?
388 407934
>>07933
А источник сигнала твой штаб не может найти, да?
15292505168900.jpg868 Кб, 1400x1009
389 407936
>>07934
Конечно не может. Это же мысленный эксперимент анона с двача, а не кого-нибудь там. А ты думал в сказку попал?
390 407937
>>07936
Больше похоже на мысленное дрочение вприсядку.
Без использования фантастических приборов для определения расстояния и параметров планет нужно наблюдение на протяжении многих орбит, а времени так понимаю не очень много.
Хотя примечательные системы типа трапписта можно на глазок узнать.
Или иные особенности, например планета с громадными кольцами и так далее.

Ещё есть вариант с ориентацией по окружающим звёздам.
391 407943
>>07933
Ты можешь примерно замерить по тому, как планета будет закрывать звезду
392 408018
>>07933
ШНН
393 408043
Венера самплер. План такой:
Щвятой хэвик запускает межпланетный аппарат к венере, в составе аппарата буксир и ландер, буксил довозит добро до орбиты, ландер отделяется и мягко садится на поверхность используя плотность атмосферы, пока выкипает охладитель - бур делает из венеры не целку и забирает образец, образец помещается в возвращаемую ебу, еба надувает аэростат, отделяется от ландера и как пробка вылетает в верхние слои атмосферы, там доразгоняется при помощи небольшого двигателя. Буксир цепляет ебу на орбите и везет прямиком на мкс, где все заражаются спидораком.
Внимание вопрос, есть ли материалы для баллона аэростата что бы выдержать ад на венерианском дне и надуться? Как поведет себя газ при таком давлении и температуре, перейдет ли он в нужное агрегатное состояние, есть ли вообще такой газ? Если нет то может ли жесткая коробка наполненная газом обеспечить необходимый подъем что бы оказаться в нужных условиях и надуть уже нормальный шар? Дохуя ли дельты можно так выиграть, ведь венера меньше земли а атмосфера больше?
394 408046
>>08043
Идея не нова, такие проекты были, но все упирается в то, что поверхность Венеры фактически нахуй никому не нужна.

https://www.jpl.nasa.gov/news/news.php?feature=5902

На Марсе хотя бы есть шансы высадить лысых макак, и даже вернуть их обратно, а Венера для людей 100% бесполезна, с тем же успехом можно высаживаться прямо в ад с чертями.

>там доразгоняется при помощи небольшого двигателя



Венера размером практически с Землю, твой «небольшой» двигатель должен выдать как минимум 7-8 км/с дельты, что автоматически означает минимальный вес ракеты в несколько тонн. Выполнимо, но нихуя не просто.

>Дохуя ли дельты можно так выиграть, ведь венера меньше земли а атмосфера больше?



Даже хуже, чем на Земле запускать с самолета, высота без скорости практически нихуя не дает.
Безымянный.png10 Кб, 499x556
395 408091
Предположим лежит себе анон на поляне и гоняет елду. Над ним пролетает спутник и останавливается до нулевой скорости прямо над аноном.
Как быстро упадет спутник на голову дрочиле? Или может он будет висеть в невесомости длительное время и анон просто свалит домой срать на двачах?
Пикрил примерная схема.
396 408094
>>08091
Ускорение епта посчитай
397 408095
>>08094
Я тупой, посчитайте за меня!
398 408098
>>08091
Дрочила находится на крутящейся планете. Спутник промажет.
399 408100
>>08095
Мне впадлу
Короче, зависит от высоты спутника
400 408116
>>08046

>Даже хуже, чем на Земле запускать с самолета, высота без скорости практически нихуя не дает.


Чому хуже? Там же на поверхности давление под 90 атмосфер, почти как в камере сгорания, как там движок будет работать? И плотность выше в 60 раз. Земное давление аж на 50 км. Выглядит так, будто без шарика там как раз не обойтись.
15296590033230.png20 Кб, 1288x556
401 408123
>>08091
А если хочешь с учетом атмосферы - то это надо серьезно так обмазывать интегралами

Можешь просто глянуть для понимания
https://www.grc.nasa.gov/www/K-12/airplane/drageq.html
https://www.grc.nasa.gov/www/K-12/airplane/airsim.html
402 408164
Аноны, Бетельгейзе может уничтожить человечество или нет?
403 408170
>>08164
Нет, ты че ебанутый что ли?
404 408204
>>07933

На глаз можно определить:
1. Класс светимости звезды (например Солнце это жёлтая звезда)
2. Цвет и общую характеристику планеты. (Например Земля это зелёно-голубая планета с атмосферой и её явлениями и большим спутником жёлтого цвета).
3. Расстояние до звезды в её диаметрах. (например Солнце на расстоянии 100 диаметров от Звезды)
4. Сделать вывод о соотношении радиуса и массы планеты по времени обращения. Время обращения легко засечь наблюдая положение светила.
5. Попытаться визуально найти другие знакомые звёзды. По угловым положениям нескольких звёзд найти будет ещё проще. (а знакомые звёзды на Млечном Пути)
6. Описать угол наклона экватора планеты относительно Млечного Пути, то есть общей плоскости эклиптики галактики.
7. Описать тепловое воздействие звезды на планету и твой корабль, по этому можно прикинуть и мощность звезды и вообще много чего ещё.
405 408205
>>08043

>Дохуя ли дельты можно так выиграть, ведь венера меньше земли а атмосфера больше?



Дохуя.

>Как поведет себя газ при таком давлении и температуре, перейдет ли он в нужное агрегатное состояние, есть ли вообще такой газ?



Нормально, газ подобрать не проблема.

>Внимание вопрос, есть ли материалы для баллона аэростата что бы выдержать ад на венерианском дне и надуться?



ХЗ, я не знаю таких пластиков. Ни фторопластов, ни кременийорганических соединений. Это большая проблема.
406 408234
>>08046
Тогда буксир сам превращается в аэростат и цепляет ебу в атмосфере а не на орбите.
возможно поднимать образец и не нужно, там ебанутая эррозия и все можно собрать из атмосферы
407 408285
Как происходит выяснение обстоятельств катастроф КА? К примеру, когда какая-нибудь маленькая трубка в движке забивается говном, из-за чего ракета падает. Как узнают, что именно эта трубка именно что забилась именно этим говном? Разве аппарат не пидорасит?
408 408288
>>08285
Ответ очень простой - в том числе и для этого там всё обмотано тыщами мильёнов самых разнообразных датчиков, передающих всё на землю в прямом эфире. Если, как в твоём примере, какая-нибудь трубка забилась, это поймут например по падению давления в той хуйне, которая получает <вещество-нейм> через эту трубку, и возможно по росту давления в той хуйне, которая в эту трубку <вещество-нейм> подаёт.
А главное, анализом аварий занимаются в том числе разработчики самой йобы - неглупые люди, которые очень хорошо знают, как работает эта техника, и примерно представляют, что могло привести к тому или иному результату (и так выяснить деталь-виновника аварии), а после этого могут представить, что могло привести эту самую деталь к отказу. Кроме телеметрии есть ещё множество видео запусков с разных камер (их реально очень много, намного больше чем видят рандомные непричастные хуи вроде нас с тобой, за стартами Шаттла камер 150 следило ЕМНИП), компьютерное моделирование и наземные испытания, в которых пытаются добиться повторения отказа.
В очень дедовые времена, когда самые дедовые деды учили Р7 и Атласы летать, такого компьютерно-телеметрического раздолья не было, и выяснять причины аварий было намного сложнее.
409 408291
>>07898
из-за приливных сил
410 408299
Есть ли открытые экзопланеты в кратных системах?
411 408300
>>08299
Да, в том числе у ближайшей к нам звезды Альфы Центавра (тройной, а точнее 2+1 системе) есть экзопланета - Проксима Центавра b называется. Более того, она находится в зоне обитаемости своей звезды и, предположительно, землеподобная. Может даже с атмосферой. Даже рептилоиды, хоть и маловкроятны, но не исключены.
412 408302
>>08300
Алсо, видисик про неё.
https://www.youtube.com/watch?v=zlP9g5nD5Lo
413 408303
>>08302
*видОсик
414 408304
>>08299
Да они вообще везде есть, даже в шаровых скоплениях. Только жизнь там не очень вероятна, ведь надо еще на синхронную орбиту встать, а также чтобы все звезды системы были стабильны.
415 408313
>>08304

>везде есть, даже в шаровых скоплениях


х2. Даже у аккрецирующих тесных двойных, даже у пульсаров. В теории даже у чёрных дыр и даже в межпланетном (и даже межгалактическом) пространстве, без гравитационной привязке к какой-либо звезде. Даже вокруг Аллаха, поговаривают, десяток-другой шариков из говна вращается.
416 408314
>>08313
фикс:

>в межпланетном


в межзвёздном пространстве, конечно же.
417 408329
Чёрные дыры существуют, или всё таки нет? Если теория относительности верна, то должны же. Стивен Хокинг им всё таки почти целую жизнь посвятил
418 408330
>>08329
Существуют, скоро даже фотографии дыры в центре галактики сделают.
419 408350
>>08330
Фотографии чего, лол? Черного пятна? ЧД обнаруживают, когда вокруг чего-то невидимого вращаются куча звезд. Или если повезет - по диску аккреции
420 408351
>>08350
Гравитационные волны же
421 408376
>>08329
Существуют объекты, которые очень хорошо согласуются с теорией ЧД по косвенным признакам.

Невозможно доказать существование ЧД, так с ней нельзя взаимодействовать электромагнитно.
Ее нельзя трогать, облучать, наблюдать и т. д.

Все доказательства существования ЧД - косвенные (гравитация, аккреционные диски, джеты).

Однако их так много и они так хорошо укладываются в теорию, что существование ЧД давно считается доказанным.
422 408386
>>08350
Ещё по возмущениям эфира можно обнаружить.
423 408391
Как можно изменить скорости выше световой?
424 408395
>>08391
Нельзя изменить то, чего не может быть
425 408396
>>08395
Блять, меня тоже твой т9 заразил, измерить
426 408398
>>08396
>>08395
Допустим, существуют некие частицы со скоростью выше световой. Как их измерить, если информацию о их движении мы будет получать со скоростью света?
427 408399
>>08164
Если бы была в сто раз ближе к Земле то в момент бабаха могла бы подогреть атмосферу на несколько градусов, ну наверное могла бы и поднять радиационный фон на порядок другой, а так пофонит месяцок и угаснет.
428 408401
>>08386
Слышал про эфир, вроде норм штука..
429 408403
>>08401
Ато. Самая передовая теория.
430 408412
>>08403
Да эт эфирный зомби наверное, ему не до теорий.
001.jpg70 Кб, 459x640
431 408419
>>04988 (OP)
Аноны, я правильно понимаю идею, что слепо дрочить на удельный импульс не совсем корректно? Ведь с ростом скорости истекающих газов проёбываемая энергия растёт быстрее тяги, и когда ракета ещё не разогналась - её полётный КПД будет низкий.

Т.е. получается, что выгоднее иметь на первой ступени короткоживущий РДТТ с низким удельным импульсом и ахуенной тягой. Второй ступенью обычный ЖРД с тягой поменьше и импульсом побольше. Третьей какой-нибудь ёба-литий-фторовый ЖРД, дающий маленькую, но очень скоростную струйку. Ну и на четвёртую ионник присобачить.

Короче, типа как на автомобильной КПП не стартовать с четвёртой, а повышать передачи по мере разгона.
432 408420
>>08419
Примерно да, но по факту идеальный вариант это керосинка на первой ступени и водород на второй, двух ступеней достаточно для орбиты в большинстве случаев, а дальше по ситуации.
Для старта важна только общая тяга, низкий импульс не обязателен на связан при текущих технологиям желательно как можно быстрее набрать вертикальную скорость, потому что каждая секунда движения вверх отнимает 10 м/с дельты.
Например, именно из-за водорода на второй ступени Сатурн мог подняться намного больше Н-1, хотя первая ступень Н-1 попизже будет.
Тот же Сатурн имел не очень хорошую тягу первой ступени 1.2 твр вроде, поэтому гравитационные потери были ужасны.
433 408422
>>08420
Ты еще забыл, что Н-1 - перетяжеленный кусок говна. С охулиардом ступеней.
434 408424
>>08420
Например, я читал про какой-то упоротый проект паровой ракеты. Суть в том, что первая ступень помимо керосинки с окислителем имеет ещё бак с обычной водой. Вода подаётся прямо в сопло и там испаряется, отъедая часть тепловой энергии. Получается штраф на скорость реактивной струи (и на удельный импульс, соответственно), но прирост тяги за счёт массового расхода.
435 408425
>>08424
Говно это и нахуй не нужно. У современных ракет проблемы только с импульсом, тяга заебись.
436 408434
>>08419
Посмотри на вторую пикчу в оп-посте. Помимо удельного импульса значение имеют конструктивное совершенство и ступенчатость. И обычно энергетическое и конструктивное совершенства в обратной зависимости. Для повышения массовой отдачи при прочих равных увеличение УИ полезно всегда, однако приходится искать наиболее выгодные соотношения по массам конструкций. А ещё для реальных изделий приходится брать во внимание стоимости и учитывать возможности разработки и производства.
Для совсем уж оптимизации размышлять не нужно, нужно решать соответствующие уравнения, учитывающие все факторы.
437 408437
Почему нельзя запилить космодром где-нибудь в Сахаре и пускать оттуда атомолеты?
IMG2822.PNG40 Кб, 564x289
438 408438
Как они узнали, как выглядит наша собственная галактика?
439 408440
>>08425

>Говно это и нахуй не нужно. У современных ракет проблемы только с импульсом, тяга заебись.


Тяги много не бывает. Больше тяги значит, что можно добавить к верхним ступеням больше топлива, заметно прибавив дельту.
440 408443
>>08440
И импульс по пизде. И тогда первая ступень будет в разы тяжелее, а значит дороже.
Лучше поставить больше двигателей.
441 408459
Когда "Венеру" расконсервируют? Уже десятилетия без новых фоточек.
И какого хуя ещё на Меркурий не садились?
Ёбаный Марс, блядь, это всё он виноват своей "землеподобностью". То, что это пыльная радиоактивная пустыня имеет хоть какую-то сраную атмосферу, является привлекательным фактором в глазах ебучих нердов и некбердов со всего мира. НУ А ТО ЧОЩ, ВОН ПОЛЕТИМ НА МАРС, ЗАПАНУЭМО!! Тупые выродки, где мои охуенные фоточки Меркурия?!
442 408461
>>08459
На Меркурий через пару месяцев полетит аппарат, правда лететь будет 7 лет. Трудно туда сесть потому что встать на орбиту нужно дохуя дельты, плюс ещё посадка, короче нескоро.
На Венеру мутные планы, вроде как роскосмос хочет, но это значит ещё дохуя лет.
443 408463
>>08459
На Марсе лендеры живут годами, на Венере — единицы часов, куда бы ты послал аппарат, если бы была возможность? Ну и дело не только в количестве научных данных, но и в их качестве и полезности.

С полезностью сразу все ясно, поверхность Венеры для людей в ближайшие десятки лет абсолютно бесполезна, жить там нельзя, добывать нечего, вода давно улетучилась в космос, вывод чего-то на орбиту сложнее, чем на Земле. Даже солнечные панели работают хуево.

В научном плане тоже не шибко много интересного. Венерианская кора тонкая и относительно молодая (возрастом в несколько сотен миллионов лет), вся история планеты давно расплавилась и утонула в мантии. Тектоники плит нет, рельеф крайне однообразный — плоские лавовые равнины покрывают практически всю планету, изредка прерываясь возвышенностями и разломами.

В отличие от того же Марса, по камням которого миллиарды лет назад текли и высыхали реки, и в грунте остались следы глобального изменения климата, на Венере в историческое время не происходило ровным счетом нихуя.

Не понимаю этого дроча на Венеру, в общем, ну хуевая же планета, даже жалко просирать научные аппараты на подобную срань. Если что-то интересное там и есть, так только атмосфера, а сама поверхность максимально скучная хуйня.
444 408464
>>08463
Марс говно без будущего. На Венере при сайфайных технологиях можно нормально жить, на Марсе никогда, только в куполах, ну то есть ничем не лучше луны.
1355943127688.jpg33 Кб, 300x300
445 408465
>>05170
У меня очень часто возникает вопрос. А почему очень много статей на научную тематику на вики, не переведено на русский? причём может быть на португальском, чешском или вьетнамском, но на русском очень часто не бывает. Это специально, чтобы школота/студентнота не могла нормально прочитать?
Вот только не надо щас начинать, что в 2018-м не знать инглиша и т.д. Есть гуглопереводчик, но сам факт отсутствия множества научных публикаций на вики на русском. Немного обидно даже как-то. Неужели всем похуй?
будущее россии.jpg52 Кб, 612x546
446 408466
>>08464

>На Венере при сайфайных технологиях можно нормально жить


Давление: 93 бар
Температура: 464 °C

Ваще изи там жить будет, курорт блять.
447 408467
>>08438
Ты когда-нибудь делал панорамную съёмку мобилкой?
448 408468
>>08464
При сайфайных технологиях на Марсе жить как раз попроще будет, чем на планете вообще без воды и с 90 лишними атмосферами газа, который некуда девать.

С Марсом рецепт прост, надо только нагреть немного и подбавить любого газа, и дальше все само пойдет, так как там все нужное для жизни есть прямо под поверхностью. Никакого правдоподобного способа убрать венерианскую атмосферу даже в фантастике еще не придумали.
449 408470
>>08459
Интерес к Венере был, когда никто не понимал, что там на поверхности.
Поэтому Союз и бомбил ее без остановки всякими "Венера №".
В 1975 году вроде бы оттуда фотки пришли (СУКА, В 1975 ГОДУ. ФОТКИ. С ВЕНЕРЫ. БЛЯДЬ, КРЕПКА БЫЛА СОВЕТСКАЯ ВЛАСТЬ)
Сейчас более менее понятно, в том числе, что там пизда и ничего, живущего больше часа туда не отправить. Поэтому экономически это нихуя не выгодно, максимум спутники ей сажать на орбиту (вроде там уже что-то узкоглазое крутится или должно вскоре).

А Марс - он и есть Марс, его относительно легко исследовать, плюс там точно была вода, что повышает интерес.
450 408471
>>08437

>Почему нельзя запилить космодром где-нибудь в Сахаре


Потому что все уже запилили там где им надо и удобно. кроме россии, у нас северный мордор, в любом случае проигрываем в полезной нагрузке.

>пускать оттуда атомолеты


Потому что атомолёты возможны только в твоём воспалённом мозгу.
451 408472
>>08468
Магнитное поле тоже обратно сможешь включить?
452 408473
>>08466
>>08468
Ну чё вы несёте?
Во первых Марс никогда не сможет удержать атмосферу. Там хуевая гравитация. Чтобы это изменить нужны такие высокие технологии, что когда они будут проще будет долететь до нормальных планет в других системах. А иначе это ничем не будет отличаться от поселений на луне, те же ебаные купола и закрытые помещения.
А концерты превращения Венеры в нормальное место придумали ещё в 70-80ых годах, например Саган предлагал много чего, причем без йоба технологий. Плюс вариант надувных объектов на высоте, там давление как на земле и лишь немногим жарче.
453 408474
>>08466
Генномодифицированные человеки там выживут.
454 408475
>>08471

> пук


Атомолеты возможны даже на технологиях 70-х, ебанашка.
455 408476
>>08472
Там оно есть по сути, особое взаимодействие с атмосферой даёт нормальную защиту. Плюс сама атмосфера блокирует значительную часть говна.
Genius.png121 Кб, 811x624
456 408477
>>08330

>скоро даже фотографии дыры в центре галактики сделают


Фотографии того, что не выпускает за свои пределы фотоны.
457 408478
>>08476
Магнитное поле в 500 раз слабее земного.
Атмосферы почти нет. И нихуя она не блокирует.
Да и что она должна по-твоему блокировать и как?

Ты здоров, друг?
458 408479
>>08463
>>08470
Запускали же аэростаты "Вега" 1 и 2. И довольно долго работали они..
459 408480
>>08477
Передадут посредством хокингова излучения.
460 408481
>>08478
Бля я думал там про Венеру. У нее взаимодействие солнечного ветра с атмосферой создает аналог небольшого, но достаточно сильного, магнитного поля.
461 408482
>>08477
Ага, а сеть EHT это наверное мошенничество?
462 408483
>>08473
Это выдуманная проблема, потеря атмосферы это сотни миллионов лет, если б на Марсе была атмосфера как на Земле, то изменение плотности за несколько тысяч лет ты бы никаким прибором не зафиксировал.

>А концерты превращения Венеры в нормальное место придумали ещё в 70-80ых годах, например Саган предлагал много чего, причем без йоба технологий.



Ну-ка, перечисли их. Дай угадаю, начинаются эти твои маняконцерты, разумеется, со скинутых в атмосферу бактерий? Я тоже такое читал, только автор не знал, ни того, что бактериям для жизни тоже вода нужна в пропорциональных количествах, чего на Венере как-то не наблюдается, ни того, что связывание углекислого газа — обратимый процесс, и вывод из цикла жизни и захоронение углерода даже на Земле идет пиздец как медленно.
463 408484
>>08479
56 минут прожил спускаемый модуль.

Bitch, please
b1e1f65e-a663-4115-bb25-982b4d50e755.jpg26 Кб, 400x507
464 408485
>>08473

>Ну чё вы несёте?


Что ты несёшь, жирдяй-каникуляр. Это уже даже не троллинг тупостью. 464 градуса блять, какие к хуям человеки на венере? Даже не учитывая давление в 90атм и дожди из расплавленных металлов.

>Марс никогда не сможет удержать атмосферу


Она там уже есть, разряженная но есть, да и похуй на неё вообще, купола решат. Герметичный домик-гараж-ракушка с системой жсо и ты сможешь капчевать там сидеть, в защите от радиации песка и ветров. А прогуляться к васяну или еотовой, в соседнюю ракушку, сможешь напялив сайфайный скафандрик в обтяжечку, с подворотами. Или по подземному тоннельчику.
465 408486
>>08475
чо же ими не пользуются?
466 408487
>>08473

>Плюс вариант надувных объектов на высоте, там давление как на земле


Да, действительно, на высоте 25км над поверхностью венеры, давление 1 атм. температура около 25-35 градусов, правда ещё в нагрузку облака из серной кислоты, и радиация жёстче чем на марсе в несколько раз. Удачного освоения, толстячок.

покинул дебильный троллячий тред
467 408489
>>08483
А где ты атмосферу возьмёшь, для начала? Посчитай сколько массы нужно для комфортного давления прежде чем нести чушь про бомбёжку полюсов.
Кстати азот где брать будешь?

>>08485
Ты очком читаешь или как? В теории Венеру можно сделать пригодной для жизни в теории, Марс только в фантазии. Зачем жить на Марсе в куполах, если можно на луне?
468 408490
>>08484

> Каждый зонд проработал около 46 часов («Вега-1»: с 2:08 UT 11 июня по 00:38 UT 13 июня; «Вега-2»: с 2:07 UT 15 июня по 00:38 UT 17 июня; указано время получения сигналов на Земле).


Кто тут у нас ещё "bitch".
469 408491
>>08487
Пруфани радиацию или чепух.
470 408492
>>08489
Двачну, Марс - оверхайпнутая хуйня для фанатов "Звёздного пути" и прочих "manbaby"-задротов.
Венера - выбор чемпионов.
471 408493
>>08487
Какой же ты неженка. Небось ещё и на EM-Ведро надрачиваешь.
472 408495
>>08490
Ты просто читать не умеешь. 56 минут отработал посадочый модуль. А 46 часов атмосферный, который с посадочного взлетел на хуй знает сколько километров вверх, где совершенно другие давление и температура.
473 408500
>>08489
Она прямо в земле лежит и в полярных шапках, суммарно на Марсе достаточно одного только летучего углекислого газа примерно на 30-60% земной атмосферы, а еще воды дохуища. Азота также есть достаточно в почве, Кьюриосити его находил. На Марсе вообще дохуя всего во вмороженном или связанном виде есть.

Отмечу, кстати, что раз уж мы сравниваем с Венерой, то сразу надо задуматься, что проще, добавить 1 атмосферу газа или убрать 90? Добавить 20-30 градусов температуры или убрать 400+? Создать защиту от солнечного ветра вблизи солнца или в два раза дальше? Плюс никаких своих концертов ты так ни разу и не назвал, хотя уже несколько раз успел заявить, какие они охуенные и реалистичные.
474 408501
>>08500
Ты, главное, скажи, как это всё сделать.

А то пиздеть - не мешки ворочать.
475 408502
>>08501
Сначала ты про Венеру хоть на один пост распиши, а то мне надоело уже писать серьезно, а в ответ слушать троллинг тупостью.
476 408503
>>08500
Бля ты цифры нужной атмосферы посчитал или хотя бы поискал?
Количество азота какое? Как его выпускать?
Плюсы Венеры только в том что там есть нормальная гравитация и атмосфера, это все.
477 408504
>>08495
Я о нём и писал, ебан.
NeptuneringsPIA02224.jpg57 Кб, 300x512
478 408506
Какие планеты и спутники могут быть потенциально хорошими кандидатами для создания человеческих колоний?
479 408507
>>08506
Это утопия, даже если бы они были, транспортировка к ним того количества грузов и людей, необходимых для существования человечества, даже в пределах СС для нынешнего физического принципа работы космических двигателей - невозможная задача.

И да - в СС таких мест нет. Через миллиард лет Солнце увеличит светимость до такой степени, что жизнь на первых четырех планетах будет невозможной в принципе.
Дальше только газовые гиганты и их крошечные спутники, ебимые приливными силами.

Смирись, ты умрешь, умрут все твои потомки и всё человечество. Затем Солнце спалит Землю, потом станет красным гигантом и окончательно сожжет, поглотив в себя Меркурий, Венеру и, возможно, Землю, а потом наступит один из предполагаемых концов Вселенной. И не будет ничего.
Поэтому live fast, die young.
480 408510
>>08438
>>08467
Добавлю про базовое черчение и построение фигуры по точкам.
481 408511
>>08507
ну в целом есть надежда на то что потомки разгадают больше законов физики чем мы и методв абузить её, еще лет 150 назад считалось что увсе в физике изучено и описано и мол мертвая наука.

Например как вариант наша вслеленная может не увеличиваться в размере а просто наблюдаемая часть заполнять пустое пространство во вселенной

Те фотоны не платят налоги за расширение вселенной и не тереяют энеригии. а может и правда все пизда
482 408513
>>08507
Ой, да вы заебали драматизировать.

Активное освоение СС будет после того, как люди наиграются с реюзом ракет и вкатятся в безракетный и частично безракетный запуск. Это тысячи тонн в сутки за копейки, даже с примитивными вакуумными трамваями в гималаях. Пара-тройка десятилетий при хорошем раскладе.

Транснептуновые и ближние межзвёздные перелёты будут после того, как завзеут ТЯРД. Ничего запредельного в нём нет, тупо постепенный прогресс сверхпроводников. 3Тл уже обыденность, 10Тл с трудом доступны. Ну а импульсный термояд на Z-машинах уже давно хуярит с положительным выходом с огромными запасом по температуре. Осталось сделать всю эту хуйню компактнее. Ожидаемый срок - начало 22 века.

Релятивистские корабли будут после того, как КПД производства антипротонов достигнет десятков процентов, а мировая энергетика сравняется хотя бы с солнечным светом, поглощаемым Меркурием. Опять же, ничего запредельного тут нет, тупо долгое и нудное масштабирование сегодняшных наработок. Срок 2-3 столетия.

Вот со сверхсветовыми всё уже сложнее, хотя зацепки есть. Пока мы их понимаем на пальцах, примерно как да Винчи понимл вертолёт. Вырастут энергопотребление, энергофокусировка и вычислительные способности порядков эдак на 8-9, тогда и поговорим о сроках.

Энивей, это не миллиарды лет нихуя. Миллиард лет можно восрать только если какое-нибудь политическое хуйло угробит на планетке высокоразвитую жизнь.
483 408520
>>08475
Заговор ракетной мафии.
484 408523
Ребята тупой вопрос, сколько стоит двигатель(установка) РД-8. Есть ли ссылка на цену?
485 408525
>>08465
Так получилось, что во второй половине XX в. английский язык стал чем-то вроде lingua academica ("язык науки", как в свое время латынь). Поэтому да, всем "похуй", т.к. азы языка выучить проще, а чтобы написать перевод более качественный, чем это умеет гуглопереводчик, нужно знать язык на достаточно высоком уровне, на что основная масса читателей научных статей (условных индусокитайцев) не имеют ни времени, ни желания.
486 408527
>>08513
Если Z-машины умеют выходить в плюс, почему итер является токамаком?
487 408550
>>08527
Потому что токамак более старая, проработанная и мейнстримная концепция, бросать которую на полпути не комильфо. Плюс он всё-таки более удобен - там просто ровно горит плазменный бублик, бери да снимай тепло.

Z-машина это такой экзотический симулятор процессов в водородной бомбе, который неожиданно выстрелил на 2млрд К. Тема эта новая, мутная, и конструктивно не очень удобная для электростанции - это надо гигантские конденсаторы быстро перезаряжать, снаряд менять с проволочками, и теплообменник туда каким-то хуем вкорячить.
А ещё распиаренный лазерный NIF поднасрал. Убили кучу бабла на эту йобу, а зажигания там так и не случилось. Что сделало чёрный пиар другим проектам импульсного термояда.

Поиск концепций при разработке чего-то принципиально нового это вообще цирк с конями. Можно в упор не замечать золотой самородок, можно выдрачивать изначально порочные вещи (и даже таки выдрочить до юзабельности), можно мерить интерес пропорционально въёбанным деньгам. Сейчас вот локхид-мартины вообще электростатический фузор из-под сукна достали и ебутся с ним.
488 408606
Зачем человеку космос если он такой слабак?
Писает, какает, не выдерживает перегрузок, радиации, мы же не сможем дальше марса улететь.
sage балтика9 489 408610
слыхал кто новость что взорвалась сверхновая и типа нам всем каюк? хотелось бы узнать будут ли губительные последствия
Без названия.png125 Кб, 1410x533
490 408618
Я не понял, так существование внеземной жизни это почти 100% факт? Т.е. не "ай вон ту билив", не "50 на 50", не идеи сумасшедших гиков и фантастов, а наша реальность?
491 408620
>>08618
Внеземная органика - факт.
Добиологические химические реакции несложных веществ, включающие в себя самоорганизацию, конкуренцию и цикличность - факт.
https://ru.wikipedia.org/wiki/Реакция_Белоусова_—_Жаботинского
Богатые тяжёлыми элементами планеты с условиями, достаточными для существования жидких растворов - факт.
Самозарождение во всей этой хуйне какого-нибудь примитивного компилятора на РНК или аналогичном "языке биопрограммирования" - очень вероятное событие.
Самозарождение клетки, на 100% питающейся подножной неорганикой типа железобактерий - вероятное событие.
А вот дальше уже много вопросов. Ибо эволюция разгоняется долго, и на переход хотя бы к ссаным трилобитам нужно убить миллиард лет. И нет никаких гарантий, что за такое огромное время на планете не случится несовместимый с жизнью катаклизм.
С разумной жизнью вопросов ещё больше.
492 408633
>>08618
Органика и органические молекулы это не значит жизнь в каком-либо виде. Это всего лишь те вещества, из которых состоит жизнь, но они вполне могут образовываться без жизни.
То есть всякая жизнь - органика, но не всякая органика жизнь.
Эта статья на картинке каким-то желтушным тоном написана. Например, на Земле метан является продуктом жизнедеятельности, а на Титане моря, реки и дожди из метана естественного происхождения.
493 408635
1) Будет ли Как-Земля находится в приливном захвате с Как-Солнцем, если не будет какой-нибудь Как-Луны?
2) Разве способная планета промёрзнуть до ядра? Там же давления вообще адовые, жарко должно быть в любом случае
494 408637
>>08635
Нет, нужно намного меньшее расстояние. У нас в системе даже маленький и одинокий Меркурий вплотную у Солнца копротивляется захвату.
Только если совсем крошечная, как астероид. Иначе давление создаст нагрев.
495 408639
>>08620
>>08633
Спасибо
496 408640
>>08635
Через 10 в нной степени лет любая планета промёрзнет до ядра, если не будет внешних аномалий.
497 408641
>>08618
Представь себе что ты заходишь на сайт и там говорят, что ты выйграл миллион единственный на всю вселенную. Ты конечно можешь предположить что это так, но каковы шансы что это так? Ответом будет ответ на парадокс ферми.
498 408663
>>08618
Ещё не 100%, вот когда джеймса уэбба запустят (постоянно откладывают, пидоры) - тогда и узнаем точно, есть ли фотосинтез на других планетах. Про обычные бактерии попытаться узнать можно, только продырявив другие планеты и спутники Солнечной системы.

Другое дело, что другая сложная жизнь может и не существовать в этой галактике. Смотря на свет от планет мы не поймём, что там: простые цианобактерии, медузы, стрекозы с два боевых истребителя размером или 10/10 няши лесные эльфиечки.

>>08635
1. Нет, даже наоборот - она бы быстрее вращалась. Солнце далеко, Земля тяжёлая, инерции у неё много.
2. Так-то должно, только за гугол лет в любом случае промёрзнет - закон сохранения массы-энергии никуда не делся.
499 408667
>>08330
С уголком, три на четыре.
500 408669
>>08610
Это в которой 200 световых лет уже другая галактика?
501 408671
>>08610

Ну видишь как тебя сейчас жарит из космоса излучением радиации? Вот это и есть каюк.
502 408672
Ребята, посоветуйте литературы про ядерные ракетные двигатели.
503 408686
Сколько проживет условный посадочный аппарат на Ио?
504 408688
>>04988 (OP)
Если поебаться в космосе - шлюха забеременеет? И каков будет плод?
505 408690
>>08663

>Другое дело, что другая сложная жизнь может и не существовать в этой галактике


Да ну нахуй, земной ты шовинист. Хотя то, почему мы не видим другой очень высокоразвитой цивилизации - загадка. Неужели в космосе нет каких-нибудь пеленгов суперразвитых?
506 408691
>>08690
Есть вариант что нам очень повезло и мы в числе первых.
507 408696
>>08506

Марс, крупные спутники Сатурна и Юпитера.
Для начала необходимо научиться сохранять биосферу Земли и вообще бережно к ней относиться. Потому что лет через 50 проблемы с экологией могут стать основными.
p0002.jpg48 Кб, 600x863
508 408705
школьник 7.jpg151 Кб, 491x500
509 408731
>>08507

>Это утопия,


Ш, ты опять выходишь на связь?
510 408734
>>08731
Тебя то кто спрашивал, псина?

И да - где перекот?
511 408739
>>08468

> Никакого правдоподобного способа убрать венерианскую атмосферу даже в фантастике еще не придумали


Пол Андерсон, "Большой дождь"
512 408745
>>08734

>перекот


Тред еще с нулевой не смыло, рачок с Энцелада.
513 408750
>>08739

>Пол Андерсон, "Большой дождь



Фантастика, нисколько не правдоподобная, и даже сомнительно научная. Ради интереса открыл, уже в самом начале появляются такие правдоподобные вещи, как натуральные демоны Максвелла.

>Трубы захватывали несущийся воздух и разделяли быстрые и медленные молекулы; более холодная фракция направлялась в систему охлаждения, поддерживающую в городе окружающую среду, обеспечивающую нормальную жизнедеятельность людей — снаружи температура превышала точку кипения воды.



Венера, опять же, не имеет ничего общего с настоящей. Основная идея рассказа — это то, что на Венере дохуища воды, просто в связанном виде, и люди пытаются ее выпустить (тот самый большой дождь). Ничего подобного, разумеется, в реальности нет: всю воду давно разбило на атомы солнечным ветром и водород сдуло в космос, в атмосфере Венеры и поверхностном грунте вода практически полностью отсутствует и сделать ее не из чего. Этот факт также автоматически отменяет бактерии, которыми в рассказе производится кислород из углекислого газа.

Про воздушные насосы и говорить смешно, ими атмосферу не переработать не то что за двадцать лет, а за двадцать тысяч бы успеть. Атмосфера, опять же, далека от настоящей, в рассказе она в основном состоит из формальдегида и других углеводородов и гораздо менее плотная, так что задача колонистов состоит больше в изменении состава атмосферы, чем в ее практически полном удалении. Никакого способа захоронить сотни квадриллионов тонн газа даже не упоминается.

Ну и плюс халявная энергия у них там повсюду — много месторождений ядерного топлива, солнце ярко светит до самой поверхности, везде дуют постоянные и сильные ветры. Ни того, ни другого, ни третьего на поверхности настоящей Венеры нет.

В общем, хуйня какая-то, для 50-х годов было норм, но после того, как АМС выяснили настоящее положение дел, читается больше как юмористическое фентези.
514 408762
>>08690
В 50 световых годах точно нет (иначе бы уже уловили и ответили), да и дальше вряд ли - ни сфер дайсона, ни искусственных черных дыр нет.

Антропный принцип. Так вышло, что в этой галактике/части галактики люди - первый разумный вид, если бы им были пеленги - уже они бы думали, почему сверхразвитых людей нет.

Даже если жизнь не редкая, ей надо 1) дохуя времени (спокойная звезда, за все время существования жизни остающаяся в спокойном районе, и спокойная планета, впрочем не слишком.) 2) дохуя энергии (жить на поверхности) 3) удача (появление митохондрий - очень маловероятная вещь, и вообще хуй знает как это случилось. В меньшей степени это применимо к многоклеточности, вторичноротости, разуму)
515 408772
>>08762
А как ты отличишь искусственную черную дыру от природной, а, мегаинженер?
516 408773
Почему Бразилия гораздо ближе к 0° меридиану, он даже проход по Бразилии, но считается что "жарко" в Нигерии, которая от него значительно севернее? Почему в Бразилии нету пустыней? Почему бразильцы не чернокожие?

Правильно ли я понимаю, что чем ближе к 0° меридиану, тем менее отчетлива смена сезонов года, а на экваторе её вообще нету? Т.к. лето, осень, весна и зима здесь почти что одинаковые? Или за счет смены сезонов в других областях формируются всякие муссоны и циклоны, которые идут к экватору и портят праздник?
517 408774
>>08773

>Т.к.


т.е /fix
518 408775
Я еще Нигер с Нигерией перепутал, но не суть, они соседи. Нигер, аазазазаз, почему они еще не поменяли название и не обиделись?
519 408776
>>08773
В Бразилии хорошо с дождями, много (относительно Нигера) пасмурных дней, значительно меньше часов солнечной активности в день (и соответственно ниже температура) и много обеспечивающей тень растительности, которая гарантировала, что безволосые обезьяны не сгорят дочерна.
В Нигере с осадками всё очень плохо, часов солнца очень много, растительности нет, облаков нет, тени нет, обезьяны торчат на солнце 24/7.

Осадки как-то связаны с физической высотой страны (бразилия едва возвышается над морем, нигер высокий) и удалённостью от тропиков (сходящиеся к экватору воздушные массы забирают воду оттуда и сбрасывают на экваторе).
1086px-EarthGlobalCirculation-ru.svg[1].png414 Кб, 1086x858
520 408778
>>08773
Общая циркуляция атмосферы такая, в Бразилию в основном дует ветер с океана, а в северную Африку с континента. На западной стороне Южной Америки, например, на той же широте пустынь дохуя, потому что вся влажность успела осесть в Андах.

Бразильцы в основном метисы, микс из португальцев и индейцев, приправленные африканскими неграми, и выглядят соответствующе, от практически 100% европейцев до натуральных нигр.
521 408780
>>08778
Что за потоки обозначаемые маленькими красными овалами расположенные выше больших красных? Есть что про них интерестного известно?
522 408781
>>08772
В искусственную срут чтобы компенсировать излучение Хокинга.
523 408784
>>08781
Ох ебать там излучение мощное, за сколько тысяч лет дыра солнечной массы килограмм высрет?
524 408787
>>08784
А если маленькая дыра и не кончается - значит срут.
Как Солнце смысла нет делать, там выхлоп слабый будет.
525 408789
Навеяно >>408783

А сколько уже денег вложили в него? Насколько реально отказаться от JWST и перенаправить деньги на другие проекты? Или сократить научную программу, выкинуть 2/3 неработающего, проблемного и смежного оборудования, поработать напильником и запустить как есть?
526 408790
>>08789
Вложили 8,8 миллиардов, еще миллиард в процессе. Реальность отказа — 0,1%, реальность запуска как есть — 0%.

Уэбб уже построен и безумные миллиарды уже потрачены, вернуть их нельзя. Запустить как есть означает немалую вероятность того, что работать он не будет вообще, а отремонтировать на орбите, как Хаббл, его не выйдет — слишком далеко.

Лишнего оборудования там почти нет, это тебе не исследователи каменюк с миллиардом разных инструментов и задач, а просто инфракрасный телескоп, только очень большой.
527 408793
>>08790
Ещё миллиард от Европы.
528 408803
Почему Оу-мяу-мяу считается первым обнаруженным межзвёздным объектом, в то время как ещё в 2015 установили, что звезда Шольца пролетала через Солнечную Систему всего несколько десятков тысяч лет назад? (doi:10.1088/2041-8205/800/1/L17)
529 408804
>>08803
Первым наблюдаемым. Существование подобных залетный камешков никогда не ставилось под сомнение.
Недавно кстати ещё одну хуйню нашли на ретроградной орбите солнца, тоже залетыш скорее всего.
530 408813
>>07904

> И вот еще вопрос. На каком расстоянии от черной дыры начинает замедлятся время?


На любом блять.
Но с разной степенью.

> свет до нас летил как бы медленнее, верно?


Нет, скорость света постоянна
531 408818
>>08813

>На любом блять.


>Но с разной степенью.


Но если подумать, гравитационные волны распространяются тоже со скоростью света. То есть, схлопнулись говна в точку - и мы узнаём что это чёрная дура только через N десятков, сотен, миллионов лет.

А если ещё сильно подумать, из-за малой скорости света Вселенная вообще обладает очень хуёвой причинно-следственной связью своих кусков друг с другом??? у меня мозг лопается от разрыва шаблона
532 408820
>>08818
Если подумать вообще заебательски - куски процессора, который обрабатывает высранное на дваче говно, тоже имеют очень хуёвую связность, т.к. слишком медленная скорость света играет заметную роль сравнительно с размерами кристалла и частотой. Так что ничего удивительного.
533 408826
>>08818
Не совсем. Например у нас уже нет причинной связи с объектами на краю наблюдаемой вселенной, но но у объектов на середине пути есть связь с объектами за границей вселенной. И так далее.
534 408827
Посоветуйте хороший учебник по дифференциальному и интегральному исчислению, надо вспомнить школьную базу и восполнить пробелы в образовании. Хорошо бы и теорию, и сборник задач с ответами и, желательно, кратким решением. Не сильно мудреную, мне не экзамены сдавать, а для общего развития + понимание предмета задачи и пути её решения + освоить практику вычислений с калькулятором.

Какой-нибудь мат.анализ для студентов с непрофильным по математике техн.образованием? Или мб курс математики для головастых 11-х классов?
image.png1,1 Мб, 900x600
535 408830
Мне нужно проконсультироваться по поводу одной штуки. Если у планеты есть кольца, то они будут видны с поверхности ночью или получится всратая тень от планеты? Или всё зависит от разных факторов? Луна так-то вроде как отражает солнечный свет, хотя с другой стороны, есть же и лунные там затмения всякие. Хз бля...
15129846333430.png482 Кб, 1366x768
536 408831
>>08830
Открой спейсдвижок, да погляди, пикрелейтед.
537 408845
>>08827
Зельдович "Высшая математика для начинающих" — лютая годнота.
538 408863
>>08845
Спасибо, гляну
539 408867
Давайте перекат уже, заебало каждый раз вниз страницу мотать.
ARYb-k6q0vU.jpg155 Кб, 1280x856
540 408961
Анон, пикрилейтед лжёт? Может ли такое количество водорода и гелия, даже разогретого в ядре до 15млн. градусов выжечь кислород и убить всё живое?
Я пробовал гуглить, но кроме ссылок на научпоп и вялых сомнений в комментариях ничего не нашёл.
542 408963
543 408964
>>08961
Ну явно подразумевается термоядерный взрыв. Расчеты надо проверять. Само по себе понятие "булавочная головка" слишком расплывачатое. Однако причем тут поглощение кислорода - на этом мои полномочия всё.
544 408965
>>08962
Благодарю
545 408996
>>08961
Какое в-во имеется ввиду?
Средняя плотность Солнца меньше плотности воды, так-то. Поправьте меня.
546 409029
>>08996
Средняя плотность таки повыше будет (на 40% больше плотности воды), но даже из самого ядра если 1 кубический сантиметр взять, нихуя не произойдет.

Это всего лишь 150 грамм водорода, пусть даже и нагретого до 15 млн градусов, и имеющего внутреннюю энергию порядка 32 гигаджоулей (~7 тонн тротила). Термоядерного взрыва никакого, разумеется, не будет (в солнечном ядре даже при ебическом давлении и так реакции очень медленно идут, тепловыделение в пересчете на килограмм там ничтожное), химической энергией тоже можно пренебречь.

На расстоянии 160 км даже не услышат ничего.
547 409033
>>09029

>160км и не услышат


Ну и похуй на эту порашу. Семь килотонн хорошо потрудятся на пользу человека.
548 409034
>>09033
Ну тонн же. Мало, пиздц. Надо больше.
549 409041
А самые большие звёзды типа чёто-там большого пса лень гуглить, у них вообще внешняя часть гравитационно связана с ядром? А то ведь так и планетарную туманность можно звездой назвать и продолжать охуевать от размеров.
550 409042
>>09041
Может быть связана, а может, и нет.

Радиус звезд обычно меряют по оптической толщине, т.е. поверхность, где фотоны перестают сталкиваться с веществом звезды и начинают свободно вылетать в космос (более точно — менее N столкновений в среднем, обычно N=1) и считается поверхностью звезды, а дальше уже корона.
551 409043
Если до завтра не перекатите, я сам перекачу, и могу сразу сказать, что вам не понравится, как я это сделаю.
552 409044
>>09042
Ну тогда хууууй знает, сверхновые должны быть самыми большими, там наверно до световых недель-месяцев такие условия сохраняются. А по-пацански надо только на главной последовательности мерить, когда со звездой нет никаких неожиданностей и есть уверенность в завтрашнем дне
201[1].jpg648 Кб, 990x767
553 409045
>>09044
С чего бы это? Сброшенная оболочка не такая плотная, рассеивается на ней совсем немного света (примерно я так почувствовал), явно не каждый второй фотон.

А по-пацански это когда у звезд вообще перестает существовать радиус? Как насчет пульсирующих звезд, где поверхность сжимается и расширяется, у них тоже радиуса нет? Хуйня какая-то, в общем, измерение по оптической толщине пусть и не идеально, зато работает на всех звездах и позволяет сравнивать их между собой.
sage 555 409174
>>08669
да хуй пойми, везде разное пишут
то 200 св. лет, то 200млн , то вообще они уже не в курсах на каком расстоянии
556 409191
>>08775
Обижаются совсем другие люди, которые даже в египте не были, не говоря уж о остальной африке. Для них же это нормальное название.
557 417512
>>08688
Такой же будет, вопрос только в развитии в условиях невесомости.
Тред утонул или удален.
Это копия, сохраненная 24 ноября 2018 года.

Скачать тред: только с превью, с превью и прикрепленными файлами.
Второй вариант может долго скачиваться. Файлы будут только в живых или недавно утонувших тредах. Подробнее

Если вам полезен архив М.Двача, пожертвуйте на оплату сервера.
« /spc/В начало тредаВеб-версияНастройки
/a//b//mu//s//vg/Все доски